Epinephrine and ACLS

Big thanks to Michelle and Adam for hosting!  Also thanks to Mitch K, Katie, Tammy, Maddy, Tim and Ethan for their erudite presentations.

Background:  Historically, the only 2 pre-hospital interventions demonstrated to improve survival with good neurologic function after OHCA have been early defibrillation and bystander CPR.  Epinephrine, used in cardiac arrest since the 1960s, has consistently been shown to improve ROSC without improving neurologic outcomes.  

There are physiologic pros and cons to epi:  alpha effects promote vasoconstriction, increasing aortic diastolic pressure and augmenting coronary blood flow and the chance of ROSC.  Beta effects can cause dysrythmias, increase myocardial O2 demand and platelet activation, and impair microvascular blood flow leading to long-term organ dysfunction of heart and brain.

 

Article #1:  Perkins GD, Ji G, Deakin CD, et al.  A Randomized Trial of Epinephrine in Out-of-Hospital Cardiac Arrest (PARAMEDIC2).  NEJM. 2018; 379: 711-721.

This is the largest randomized data set on the use of epinephrine in OHCA.  In this pragmatic RCT from the UK and 5 National Health Service ambulance services, a total of 8,014 patients received 1 mg epinephrine every 3-5 minutes vs. placebo for OHCA.    Groups were well balanced with regards to baseline characteristics.

The primary outcome was 30-day survival.  Secondary outcomes included survival until hospital discharge with a favorable neurologic outcome (modified Rankin scale, mRS ≤ 3).  There was a large difference in achieving ROSC favoring epi (36% vs 12%), and at 30 days, significantly higher overall survival in epi patients (3.2% vs 2.4%).  However at hospital discharge the same percentage of patients had favorable neuro outcome in both groups (2.2% epi, 1.9% placebo), and more survivors in the epi group had severe neurologic outcomes, mRS 4 or 5 (31% vs 18%).

Limitations: 

--Median time from EMS call to epinephrine = 20 minutes (into “metabolic phase” of cardiac arrest).  Early epi has been associated with improved outcomes. 

--All centers were in the UK, limiting external validity. 

--Low overall survival, although this is partially explained by exclusion of >600 patients with ROSC immediately after defibrillation and therefore never needed epi.

--Hospital care was not standardized, although national guidelines were followed. 

--No data were reported on baseline neurologic status of patients.

Conclusion:  Epinephrine saves bodies, but not brains.

 

Article #2:  Funada A, Goto Y, Tada H et al.  Effects of Prehospital Epinephrine Administration on Neurologically Intact Survival in Bystander Witnessed Out-of-Hospital Cardiac Arrest Patients with Non-Shockable Rhythm Depend on Prehospital Cardiopulmonary Resuscitation Duration Required to Hospital Arrival.  Heart and Vessels. 2018

 

Observational trial in Japan of 118,000 patients with bystander witnessed OHCA and non-shockable rhythm, with intent of determining time-dependent effects of pre-hospital epinephrine administration.   Due to the organization of the EMS system in Japan, some patients never received pre-hospital epi, and outcomes for this group were based on duration of time from CPR initiation by EMS to hospital arrival.   The primary outcome was 1-month survival with good neurologic outcome (cerebral performance category 1 or 2).  There was a lot of slicing and dicing, but bottom line was an association of better outcomes if short transport time to the hospital or early epinephrine in the field.  The study didn’t include any data on the number of patients with poor neurologic outcomes, and is an observational trial with all limitations intrinsic to a retrospective data set.     

Conclusion:  Early epi was associated with improved neuro outcomes in witnessed OHCA with non-shockable rhythm in this retrospective observational study.

 

Article #3:  Belletti A, Benedetto U, Putzu A et al.  Vasopresors During Cardiopulmonary Resuscitation.  A Network Meta-Analysis of Randomized Trials.  Critical Care Medicine. 2018; 46(5) e443-e451.

In this network meta-analysis (statistical approach to compare multiple interventions/treatments), 28 RCTs of nearly 15,000 adults with either IHCA or OHCA receiving vasopressors were evaluated to identify preferred vasopressor for the outcomes of ROSC, survival, and survival with good neurologic outcome.  PARAMEDIC2 was not included due to timing of publication.  Only a combination of epinephrine, vasopressin, and methylprednisolone was associated with increased likelihood of ROSC, and survival with good neurologic outcome.   This therapy combination was evaluated in 2 studies in patients with IHCA.  When analyzing OHCA arrest studies, no treatment was superior over others.  When evaluating shockable rhythm and non-shockable rhythm separately, again there was no vasopressor treatment superior to another.  There is no significant randomized evidence to support nor discourage the use of epinephrine during cardiac arrest.

 

Future of epi?  Don’t expect a change in the AHA ACLS recommendations in 2020.  Benefits of epinephrine may be time dependent (early is better), rhythm dependent (works better in non-shockable rhythms), and dose-dependent (less may be more).   If epi was a new drug being evaluated for use in cardiac arrest, it might be a hard sell, but epi is stocked on EMS rigs around the world, and there are ongoing research efforts to determine how to optimize its use and gain both circulatory and clinical improvement.  For example, look for results from a large Korean RCT after 2020 evaluating effect of vasopressin/epinephrine/steroids in OHCA:  https://clinicaltrials.gov/ct2/show/NCT03317197

 

 

 

September 2017




Thursday, September 21, 2017

To Treat or Not to Treat:  Muscle Relaxants in Back Pain; Steroids in Pharyngitis; Aggressive IVF in Pancreatitis

Many thanks to Mike and Sarah Marynowski for hosting and to Matt DeStefani, Elisa Wing, Maddy Hawkins, Huu Tran, Jenny Denk and Jeff Florek for their outstanding analyses!
 

Article 1:  Friedman BW, et al: Diazepam Is No Better Than Placebo When Added to Naproxen for Low Back Pain. Ann Emerg Med 2017;70:169-176. 
In this double blind RCT, 114 patients with non-radicular, atraumatic acute low back pain without “red flag symptoms” treated at one urban health care system were either treated with naprosyn + diazepam or naprosyn + placebo.  All patients received a teaching session about care of low back pain and were discharged from the ED.  Seventy percent of patients in the diazepam group did take the diazepam one or more times daily.  Primary outcome was improvement on a validated functional impairment survey, Roland Morris Disability Questionnaire (RMDQ), at one week.  Secondary outcomes included pain intensity at 1 week and 3 months, frequency of pain, and frequency of analgesic use.  A strength of this study was the use of these patient centered outcomes. 
Results:   Functional improvement as measured by the RMDQ at one week was exactly the same between the 2 groups.   There were also no significant differences in the secondary outcomes, although trends favored placebo for both longer term improvement as well as for adverse events.
Limitations: There were a number of exclusion criteria, limiting generalizability.  Results were susceptible to recall bias, as patients had to think back to prior symptoms.  Dosing schedule was not rigid, although this mirrors real life. 
Practice changing?  Yes, for Elisa and Matt.  It would be nice to have included a reliable assessment for muscle spasm, but the vast majority of these patients were felt to have muscle spasm by the practitioners.  Unfortunately prior studies have demonstrated limited utility for other analgesics in treating low back pain.  Yoga may help, acupuncture may help, and for uncomplicated low back pain, time helps; at 3 months 90% of patients had resolved pain. 
Bottom line:  Given the results of this and other prior studies, and the concern for impairment/abuse from using diazepam, only a minimal number of physicians at JC will plan to prescribe diazepam to this patient population.  At best, they will reserve diazepam for severe spasm, and associated insomnia/anxiety.
 
Article 2:  Hayward GN, et al: Effect of Oral Dexamethasone Without Immediate Antibiotics vs. Placebo on Acute Sore Throat in Adults: A Randomized Clinical Trial. JAMA. 20 I 7;8(317)1535-1543. 
In this double blind RCT of 565 adult patients with acute pharyngitis deemed not to require immediate antibiotics and treated at 42 family practice sites in England, patients received either one dose of 10 mg oral dexamethasone or placebo.  Primary outcome was complete resolution of symptoms at 24 hours.  Secondary outcomes included complete resolution of symptoms at 48 hours, duration of moderately bad symptoms, and suppurative complications.  The majority of patients did use OTC analgesics for symptom control.  Rapid strep testing was not available, although 15% and 19% of patients in the 2 groups ended up with positive Strep cultures.  Although complete resolution of symptoms at 24 hours may seem an unrealistic primary outcome, it has been used before and facilitated a high response rate.
Results:  For the primary outcome, at 24 hours 23% of the dex group compared with 18% of placebo group achieved complete resolution of symptoms (RR 1.28, NNT = 20), which was not statistically significant.  At 48 hours, there was a significant difference (35% in dex vs 27% in placebo) in complete resolution of symptoms (RR 1.31, NNT = 12).  Results were similar for the 40% of patients offered a “delayed” antibiotic prescription vs. those not offered delayed antibiotics.  There were no significant differences in any other secondary outcomes.
Limitations:  The study was underpowered to detect differences in adverse outcomes.  Potential adverse effects of increased steroid prescribing are uncertain.  Patients in this study were young with good access to primary care.   
Practice Changing?  Huu and Maddy are not sold.  While prior studies have demonstrated very positive effects for dexamethasone in patients with pharyngitis also treated with antibiotics, results in this trial of less severe pharyngitis were more modest. 
Bottom line:  Although the JC crowd would like to be able to offer additional symptom control to our patients with pharyngitis regardless of severity, indiscriminate use of dexamethasone was not supported, and most plan to reserve steroids for severe pharyngitis.
 
Article 3:  Buxbaun JL, et al: Early Aggressive Hydration Hastens Clinical Improvement in Mild Acute Pancreatitis. Am J Gastroenterol 2017;112:797-803. 
In this randomized trial of 60 patients with acute mild pancreatitis (no SIRS or organ failure), half received early “aggressive” IVF using LR with 20 cc/kg bolus followed by 3 cc/kg/hour vs. “standard” IVF with 10 cc/kg bolus followed by 1.5 cc/kg/hour.  Patients were assessed at 12 hour intervals, with fluid adjustments and po status decisions based on HCT, BUN, creatinine, and pain level.  Primary outcome was a combined endpoint at 36 hours of decreased HCT, BUN, creatinine; improved pain; and tolerance of oral diet.  Secondary outcomes included the rate of clinical improvement, development of SIRS, development of severe pancreatitis, and volume overload (defined by physical exam).
Results:  Significantly more patients in the aggressive fluids group had clinical improvement at 36 hours (70% vs. 42%, p=0.03, NNT=4), and persistent SIRS occurred less commonly with aggressive fluids (7.4% vs. 21%; OR=0.12, 0.02-0.94; NNT=8).  The rate of clinical improvement was significantly greater in the aggressive IVF group.
Limitations:  Although no patients in the study developed signs of fluid overload, patients at risk for volume overload were excluded, limiting external validity.  The composite outcome has not been validated.  Patients overall were relatively young and healthy.  It’s a really small study; persistent SIRS developed in 2 vs. 7 patients.
Practice Changing?  This is not a high enough quality study to be practice changing per Jenny.  Jeff was more willing to follow the paper’s recommendations.  However, studies on severe pancreatititis have demonstrated negative outcomes associated with aggressive IVF; third spacing, ARDS, renal failure, CHF, abdominal compartment syndrome:  aggressive IVF treatment should be reserved for patients with mild pancreatitis.
Bottom line:  Overall, the majority at JC felt aggressive IVF are appropriate for patients with mild pancreatitis, and do align with current American College of Gastroenterology recommendations for fluids in pancreatitis.
 
EOL
 
 

 

July Journal Club

Journal Club Thursday July 13, 2017.  Contrast Induced Nephropathy.

Thank you to Harwood for an evening of pastoral scenery and champagne cocktails, and to Marshalla, Graeme, Stanek, Ashley, Oyin, and Anita for their pithy analyses.

Background:  Acute kidney injury (AKI) is associated with overall worse short-term outcomes including dialysis and death.  Uncontrolled studies have suggested IV iodinated contrast material is a risk for contrast material induced nephropathy (CIN), a form of AKI.   Therefore, contrast is a risk for dialysis and death, and should be avoided in patients with pre-existing/past renal dysfunction, or with other high-risk conditions such as diabetes, HIV, HTN, and CHF.      

Or maybe not...

 

Article 1:  McDonald RJ, McDonald JS, Carter RE et al.  Intravenous contrast material exposure is not an independent risk factor for dialysis or mortality.  Radiology. 2014;273:714-725.

In this retrospective single center study from Mayo, 10,673 patients receiving contrast (C) for abdominal/pelvic/chest CTs were propensity score matched with an additional 10,673 patients who received non-contrast (NC) CTs, to compare outcomes of AKI, 30 day dialysis, and 30 day mortality.   AKI developed in 4.8% (C) vs. 5.1% (NC), dialysis in 0.2% (C) vs. 0.3% (NC), and mortality in 8% (C) vs. 9% (NC).   In “high risk” groups with diabetes, CHF, or acute/chronic renal failure, risks of dialysis and death were higher, but there were no significant outcome differences in high risk patients who received contrast vs. non-contrast CTs.  Yes, if AKI developed, rates of dialysis and death were higher, and rates of dialysis and death were higher in patients with poor baseline renal function, but contrast material exposure was not an independent risk factor for these bad outcomes.

Limitations:  It’s a single center retrospective study.  Unknown why certain high risk patients were chosen to receive contrast.  Causes of mortality are not identified.  Although it’s controversial if anything helps for nephroprotection, authors did not identify what agents (NS, bicarb, NAC) were used or how often.  And, although a large study, only 958 patients with baseline creatinine ≥ 2.0 received contrast.

Bottom line:  IV contrast material for CT scans was not associated with increased risk of AKI, dialysis, or death, even in traditionally high-risk patients.

 

Article 2:  McDonald RJ, McDonald JS, et al.  Risk of Acute Kidney Injury, Dialysis and Mortality in Chronic Kidney Disease Patients following Intravenous Contrast Material Exposure. Mayo Clinic Proc. 2015;90:1046-1053.

Again from Mayo, a retrospective look at 1220 patients with chronic kidney disease (CKD) Stage III (GFR 30-59) and 491 patients with CKD Stage IV/V (GFR < 30), all receiving IV contrast CTs and propensity score matched with patients receiving non contrast CTs.  Again, rates of AKI, 30 day dialysis, and 30 day mortality were compared between contrast and non-contrast groups.  Many of these patients were included in prior work by McDonald.  For this study, they performed full chart reviews and included a more comprehensive list of clinical variables in the propensity score model in an attempt to improve the strength and accuracy of the conclusions.   For all stages of CKD and across all outcomes, there were no significant differences between groups.  Stage 3 CKD group, AKI 10% (C) vs. 15% (NC); dialysis 0.4% (C) vs. 0.4% (NC); mortality 9% (C) vs. 11% (NC).  In the stage 4/5 CKD group, AKI 21% (C) vs. 20% (NC); HD 1.7% (C) vs. 0.7% (NC); death 18% (C) vs. 19% (NC).

 

Additional sensitivity analysis evaluating patients with stable pre-CT kidney function, and comparing effects of IV fluid administration at the time of CT were performed, again demonstrated no significant outcome differences between contrast and non-contrast groups.

Limitations reflect the single center, retrospective nature of the trial.

Bottom line:  Specifically in patients with CKD, IV contrast was not associated with increased risk of AKI, 30 day dialysis, or 30 day mortality.

 

Statistics Pearl:

What’s this Propensity Score Matching (PSM) thing?

PSM is a statistical technique used in observational studies as a way to more closely approach the strength of a randomized control trial.  It attempts to approximate randomization by matching populations from large databases according to patient characteristics, risk factors, and co-morbidities.   This minimizes bias associated with treatment assignments, however PSM only accounts for measured/observable covariates/confounders.  Hidden bias is therefore still likely.   PSM also requires large patient populations.

 

 

Article 3:  Hinson, JS et al: Risk of Acute Kidney Injury After Intravenous Contrast Media Administration. Ann Emerg Med 2017;69:577-586.

 

From Johns Hopkins, a single-center retrospective analysis of nearly 18,000 adult ED patients with baseline creatinine < 4.0, cohorted into three groups:  CT with contrast, CT without contrast, and similar patients without CT.

Primary outcome was incidence of AKI, and secondary outcomes were development of new CKD, dialysis, and renal transplant, all at 6 months. 

Rates of AKI were similar: 10.6% (with contrast) vs. 10.2% (no contrast) vs. 10.9% (no CT).  IV contrast was also not associated with increased risk of the secondary outcomes of CKD, dialysis, or renal transplant at 6 months.

 

Authors did examine treatment decisions including administration of IV contrast and the administration of IV fluids.  Physicians were less likely to use IV contrast in patients with decreased renal function, and more likely to administer IV fluids when using IV contrast.

Limitations:  Another single center retrospective study.  Most patients were admitted to the hospital, reflecting higher acuity, and may overestimate the incidence of AKI in the general ED population.  Most patients had pre-test creatinine ≤ 1.4.

Bottom line:  IV contrast administration was not associated with increased incidence of AKI, new CKD, need for dialysis or need for renal transplant.

 

Poll at the end of the evening:  after evaluating these studies, overall attendings feeling more comfortable ordering needed CT scans with IV contrast on patients with GFRs down to around 35.  For potential life threatening diagnoses, attendings consistently support giving contrast (trauma, dissection).   Protection?  Data are all over the place, but most attendings will give IVF. 

Journal Club bottom line:  Understanding of CIN has been complicated by reliance on studies predating the use of low/iso-ismolar contrast, and extrapolation from arterial angiographic studies.  These current 3 well done studies challenge the notion that IV contrast administration for CT scans is a risk for AKI, emergent dialysis, and short term mortality.  A large prospective study is required to definitively end this controversy,  but this literature supports using IV contrast when medically indicated, even in patients with chronic kidney disease, diabetes, and CHF.

 

 

eol

 

 

 

 

 

 

 

July 2017

Journal Club Thursday July 13, 2017.  Contrast Induced Nephropathy.

Thank you to Harwood for an evening of pastoral scenery and champagne cocktails, and to Marshalla, Graeme, Stanek, Ashley, Oyin, and Anita for their pithy analyses.

Background:  Acute kidney injury (AKI) is associated with overall worse short-term outcomes including dialysis and death.  Uncontrolled studies have suggested IV iodinated contrast material is a risk for contrast material induced nephropathy (CIN), a form of AKI.   Therefore, contrast is a risk for dialysis and death, and should be avoided in patients with pre-existing/past renal dysfunction, or with other high-risk conditions such as diabetes, HIV, HTN, and CHF.      

Or maybe not...

 

Article 1:  McDonald RJ, McDonald JS, Carter RE et al.  Intravenous contrast material exposure is not an independent risk factor for dialysis or mortality.  Radiology. 2014;273:714-725.

In this retrospective single center study from Mayo, 10,673 patients receiving contrast (C) for abdominal/pelvic/chest CTs were propensity score matched with an additional 10,673 patients who received non-contrast (NC) CTs, to compare outcomes of AKI, 30 day dialysis, and 30 day mortality.   AKI developed in 4.8% (C) vs. 5.1% (NC), dialysis in 0.2% (C) vs. 0.3% (NC), and mortality in 8% (C) vs. 9% (NC).   In “high risk” groups with diabetes, CHF, or acute/chronic renal failure, risks of dialysis and death were higher, but there were no significant outcome differences in high risk patients who received contrast vs. non-contrast CTs.  Yes, if AKI developed, rates of dialysis and death were higher, and rates of dialysis and death were higher in patients with poor baseline renal function, but contrast material exposure was not an independent risk factor for these bad outcomes.

Limitations:  It’s a single center retrospective study.  Unknown why certain high risk patients were chosen to receive contrast.  Causes of mortality are not identified.  Although it’s controversial if anything helps for nephroprotection, authors did not identify what agents (NS, bicarb, NAC) were used or how often.  And, although a large study, only 958 patients with baseline creatinine ≥ 2.0 received contrast.

Bottom line:  IV contrast material for CT scans was not associated with increased risk of AKI, dialysis, or death, even in traditionally high-risk patients.

 

Article 2:  McDonald RJ, McDonald JS, et al.  Risk of Acute Kidney Injury, Dialysis and Mortality in Chronic Kidney Disease Patients following Intravenous Contrast Material Exposure. Mayo Clinic Proc. 2015;90:1046-1053.

Again from Mayo, a retrospective look at 1220 patients with chronic kidney disease (CKD) Stage III (GFR 30-59) and 491 patients with CKD Stage IV/V (GFR < 30), all receiving IV contrast CTs and propensity score matched with patients receiving non contrast CTs.  Again, rates of AKI, 30 day dialysis, and 30 day mortality were compared between contrast and non-contrast groups.  Many of these patients were included in prior work by McDonald.  For this study, they performed full chart reviews and included a more comprehensive list of clinical variables in the propensity score model in an attempt to improve the strength and accuracy of the conclusions.   For all stages of CKD and across all outcomes, there were no significant differences between groups.  Stage 3 CKD group, AKI 10% (C) vs. 15% (NC); dialysis 0.4% (C) vs. 0.4% (NC); mortality 9% (C) vs. 11% (NC).  In the stage 4/5 CKD group, AKI 21% (C) vs. 20% (NC); HD 1.7% (C) vs. 0.7% (NC); death 18% (C) vs. 19% (NC).

 

Additional sensitivity analysis evaluating patients with stable pre-CT kidney function, and comparing effects of IV fluid administration at the time of CT were performed, again demonstrated no significant outcome differences between contrast and non-contrast groups.

Limitations reflect the single center, retrospective nature of the trial.

Bottom line:  Specifically in patients with CKD, IV contrast was not associated with increased risk of AKI, 30 day dialysis, or 30 day mortality.

 

Statistics Pearl:

What’s this Propensity Score Matching (PSM) thing?

PSM is a statistical technique used in observational studies as a way to more closely approach the strength of a randomized control trial.  It attempts to approximate randomization by matching populations from large databases according to patient characteristics, risk factors, and co-morbidities.   This minimizes bias associated with treatment assignments, however PSM only accounts for measured/observable covariates/confounders.  Hidden bias is therefore still likely.   PSM also requires large patient populations.

 

 

Article 3:  Hinson, JS et al: Risk of Acute Kidney Injury After Intravenous Contrast Media Administration. Ann Emerg Med 2017;69:577-586.

 

From Johns Hopkins, a single-center retrospective analysis of nearly 18,000 adult ED patients with baseline creatinine < 4.0, cohorted into three groups:  CT with contrast, CT without contrast, and similar patients without CT.

Primary outcome was incidence of AKI, and secondary outcomes were development of new CKD, dialysis, and renal transplant, all at 6 months. 

Rates of AKI were similar: 10.6% (with contrast) vs. 10.2% (no contrast) vs. 10.9% (no CT).  IV contrast was also not associated with increased risk of the secondary outcomes of CKD, dialysis, or renal transplant at 6 months.

 

Authors did examine treatment decisions including administration of IV contrast and the administration of IV fluids.  Physicians were less likely to use IV contrast in patients with decreased renal function, and more likely to administer IV fluids when using IV contrast.

Limitations:  Another single center retrospective study.  Most patients were admitted to the hospital, reflecting higher acuity, and may overestimate the incidence of AKI in the general ED population.  Most patients had pre-test creatinine ≤ 1.4.

Bottom line:  IV contrast administration was not associated with increased incidence of AKI, new CKD, need for dialysis or need for renal transplant.

 

Poll at the end of the evening:  after evaluating these studies, overall attendings feeling more comfortable ordering needed CT scans with IV contrast on patients with GFRs down to around 35.  For potential life threatening diagnoses, attendings consistently support giving contrast (trauma, dissection).   Protection?  Data are all over the place, but most attendings will give IVF. 

Journal Club bottom line:  Understanding of CIN has been complicated by reliance on studies predating the use of low/iso-ismolar contrast, and extrapolation from arterial angiographic studies.  These current 3 well done studies challenge the notion that IV contrast administration for CT scans is a risk for AKI, emergent dialysis, and short term mortality.  A large prospective study is required to definitively end this controversy,  but this literature supports using IV contrast when medically indicated, even in patients with chronic kidney disease, diabetes, and CHF.

 

 

eol

 

 

 

 

 

Bystander Naloxone for Opioid Overdose

November 3, 2016

Background:  In the USA, overdose has surpassed MVCs as the leading cause of injury death with approximately 43,000 deaths in 2013.  Over 80% of these were unintentional, and 37% involved opioids.

One option for intervention is to provide naloxone rescue kits to drug users and their friends/families.  This approach has been endorsed by the WHO, the American Public Health Association, state legislatures and public health departments.

Arguments against:  Will users consider naloxone a “safety net” and choose to actually increase their drug use?  Can drug users and community members be trusted to safely respond to overdoses?  These articles are representative of the data responding to these questions.

 

Article 1.  Doe-Simkins M, Quinn E et al.  Overdose rescues by trained and untrained participants and change in opioid use among substance-using participants in overdose education and naloxone distribution programs:  a restrospective cohort study.  BMC Public Health 2014. 14:297.

This retrospective cohort study from 2006-2010 examines the impact of a well-established Massachusetts based initiative consisting of 8 agencies that provide overdose education and naloxone distribution (OEND).  Authors aimed to compare success of rescues from trained vs. untrained bystanders, and to determine if naloxone distribution changed heroin use.

“Trained” participants received training from the program, and “untrained” participants were contacts of the trained participants, so should have been exposed to some information about OEND.

Authors reviewed results of 599 rescue reports from 4,926 substance using participants (295 trained and 78 untrained).  There were no significant differences in help-seeking, rescue breathing, staying with the victim or in the success of naloxone administration between trained and untrained participants.  There were also no significant changes in the use of heroin when comparing use over at least 30 days in drug users who presented to different centers over time, although more participants increased rather than decreased benzodiazepine and barbituate use.

Although technically a negative study with regards to differences in overdose rescue management between trained and untrained participants, the authors conclude that as there are no significant differences in behaviors overall between these groups and there is no significant increase in heroin use, the results support an expansion of OEND programs with naloxone potentially becoming an over the counter medication.  

Limitations?  No systematic collection of data, potential bias that participants may provide incomplete or selected information that they feel investigators want to hear.  Other societal confounders may have affected drug use patterns over the study period.

Bottom line from discussants: OEND programs are likely making meaningful impact in communities, although gathering data to prove this is challenging.  

 

Article 2:  Giglio RE, Guohua L, DiMaggio CJ et al.  Effectiveness of bystander naloxone administration and overdose education programs:  a meta-analysis. Injury Epidemiology 2015. 2:10.

This meta-analysis of the literature on the effectiveness of OEND included 9 articles using pre-defined criteria with the aims of 1) evaluating odds of recovery after overdose and 2) impact of training on overdose recognition and knowledge of management of overdose.

Results:  bystander naloxone administration was associated with a significantly increased odds of recovery from overdose compared to no naloxone administration (OR = 8.58, 95% CI = 3.90 to 13.25).  Training led to improved knowledge regarding overdose recognition and management (standardized mean difference = 1.35, 95% CI = 0.92 to 1.77).

Limitations:  The meta-analysis did not include a robust discussion of study quality.  Outcomes of the relatively small number of witnessed overdoses (n=66) was largely based on self-report.  There was lots of study heterogeneity, and the pre/post knowledge data were limited with very short term follow-up of knowledge retention.  Three studies did not report duration of training; details of training curriculum and evaluation tools were also lacking.

Bottom line from discussants:  education/training may not be very helpful, butbystander naloxone itself is life saving.

 

Article 3:  McDonald R, Strang J.  Are take-home naloxone programmes effective ?  Systematic review utilizing application of the Bradford Hill criteria. Addiction 2016;111:1177-1187.

Aims:  to review the impact and safety of take-home naloxone.

This systematic review used narrative synthesis of 22 observational studies to assess the effects of take-home naloxone programs on overdose mortality as well as take-home naloxone safety.  Bradford Hill criteria are used in public health to assess causality when only observational data are available, and include the following 9 factors:  strength of association, consistency, specificity, temporality, dose-response relationship, plausibility, coherence, experimental evidence and analogy.   The authors determined that the included studies met all of these criteria, supporting the conclusions that take-home naloxone reduces overdose related mortality with a low rate of adverse effects.  

Limitations:  studies again rely on self report, there is a lack of systematic followup, a high level of study dropout, selection for participants with good experiences (survived), details of training are limited, and the most important Bradford Hill criterion, experimental evidence, was only represented by one study that used a quasi-experimental research design.

Bottom line from discussants: again, poor quality data which are overwhelming in favor of naloxone distribution and bystander use.

 

What’s the downside?  In Illinois there are no legal ramifications for physicians to prescribe naloxone.  Although data are never going to be high quality for this type of public health intervention, results consistently demonstrate positive outcomes and the potential to save many lives.  Naloxone is safe, with the most common side effect being precipitation of opioid withdrawal.  Opioid overdose kills, and as Christian stated, can also cause tremendous morbidity from respiratory depression and brain hypoxia.  

Analogy of epipens:  we prescribe potentially dangerous medication without thinking twice, studies have shown only about 10% of epipen users use it correctly.  Analogy of CPR instruction:  years may go by without bystanders doing CPR on a cardiac arrest patient, but heroin user has a high likelihood of being at the site of an overdose.

October 2016 EMRAP also discussed that naloxone may save not only the patient in front of you, but their support system:  friends, family, other users, as heroin is often used socially in groups....trickle down effect.  

What is available in Illinois?   Individuals can go to Marianos or Walgreens and obtain intranasal naloxone without a prescription, along with 1:1 private counseling from a pharmacist.  Cost approximately $140 at Marianos, Walgreen’s about 80$.  Covered by many insurance plans.  Also coupon available online for Ezvio (www.ezvio.com), the$$$$ talking naloxone auto-injector.  A cheap alternative-Mike Kennedy: price of 0.4 mg vial of naloxone 10$.

Liz Regan:  “Lali’s Law” PA99-0480, passed in 2015 in Illinois, named after a teenager who died from opioid/benzo overdose, allows trained pharmacists to dispense naloxone to individuals at risk of opioid overdose, their friends/family, first responders, and school nurses.  This comprehensive reform bill also provides criminal immunity for health care professionals who prescribe and dispense naloxone pursuant to the law, and protection from civil liability for both pharmacists who dispense naloxone and laypersons who administer naloxone in good faith.  http://www.ilga.gov/legislation/publicacts/99/PDF/099-0480.pdf

At ACMC?  Michael Cirone and Kyle Bernard will be speaking with hospital administration regarding training interested PharmDs, physicians and nurses to dispense naloxone kits obtained from the Chicago Recovery Alliance (anypositivechange.org).  Kudos to both for moving this important effort forward.

Ted:  consider tailoring discharge instructions to patient’s insurance, and use Medical Social worker.  Harwood and McKean:  this is an opportunity for institutional control/policy; standardize treatment so patients leave informed and with naloxone in pocket.  Overcome the barriers to care.

 

 

ICH and TBI

July 14, 2016

Journal Club Synopsis:  ICH and TBI:  Challenging Standard Management

A big thank you to Harwood for another peaceful summer escape to the marshlands and forest.  Also big thanks to presenters Patrick Holland, Dan DeWeert, Jenny Denk, Dan Nejak, Nate West (barbarian), and Mike Stanek (wizard).

1.      Qureshi AI, et al: Intensive Blood-Pressure Lowering in Patients with Acute Cerebral Hemorrhage. N Engl J Med  2016 June 8.

Although evidence has suggested that hematoma expansion and morbidity/mortality in hypertensive patients with ICH might be reduced with early reduction in SBP, there has been no consensus for blood pressure targets in these patients.  In this RCT of 1000 adult patients with ICH and elevated BP presenting to 110 international sites (including ACMC; go E. Kulstad and Stanek!), 500 patients were randomized to a SBP target of 110-139 mm Hg (intensive treatment) and 500 to a SBP target of 140-179 mm Hg (standard treatment).  Inclusion criteria were at least one initial SBP of ≥ 180, supratentorial ICH with hemorrhage volume < 60 cm3, GCS ≥ 5, and initiation of antihypertensive treatment within 4.5 hours after symptom onset.  Primary outcome was death or disability defined as modified Rankin scale score of 4-6 at 3 months.  Secondary outcomes included scores on a health state index and ICH expansion at 24 hours.  Serious adverse events and safety outcomes including neurologic deterioration and renal adverse events were also recorded.  Nicardipine was used as the first-line antihypertensive, and if needed, labetolol was the preferred second-line agent.

Results:  The primary outcome of death or disability was observed in 39% of the intensive-therapy group, and in 38% of the standard-treatment group (no significant difference).  There were no significant differences in health state index scores.  Interestingly, although there was a trend towards more expansion of ICH in the standard treatment group, this did not translate to higher morbidity/mortality for these patients.  The rate of serious adverse events and early neurologic deterioration was higher in the intensive treatment group.  There was a significantly higher rate of renal adverse events within 7 days in the intensive treatment group (9% intensive, 4% standard). 

Systolic blood pressure was relatively tightly controlled for both groups, resulting in a study that really compared SBP of 140s to SBP of 120s.  It is therefore unclear if a maintaining a higher SBP, e.g. SBP 160-170, would result in the same outcomes.  Points were also made that the majority of patients presented with a GCS of 15, limiting the external validity in a sicker patient population, and that the best goal may be a smooth maintenance of blood pressure, rather than a forced low target that results in significant SBP variability.  Not surprisingly, there were many more treatment failures (inability to achieve/maintain target SBP) in the intensive treatment group.  There was a trend favoring intensive treatment in patients presenting with low GCS scores (3-11).

Bottom line:  a target SBP of 140s resulted in similar death/disability compared to a target SBP of 120s in patients with relatively small volume ICH and overall high initial GCS scores.  Tight SBP control was associated with a trend toward more serious adverse events as well as a significantly higher rate of adverse renal events.  Aggressive reduction of SBP in patients with ICP does not demonstrate a death/disability benefit and is associated with increased incidence of serious adverse events. 

2.      Baharoglu MI, et al:  Platelet transfusion versus standard care after acute stroke due to spontaneous cerebral haemorrhage associated with antiplatelet therapy (PATCH): a randomised, open-label, phase 3 trial.  Lancet  2016 25;387:2605-2613.

Ok, so aggressive BP control doesn’t work in ICH.  How about platelet transfusion for patients taking anti-platelet agents?  People taking anti-platelet agents have increased mortality after ICH, and it’s intuitively attractive to transfuse platelets in this population.  In this small study (41 sites, 6 years...190 patients-ouch!), adults presenting within 6 hours of symptom onset after supratentorial non-traumatic ICH symptom with a GCS score of ≥ 8, who had received anti-platelet therapy for at least 7 days prior, were randomized open label to either receiving standard care or standard care + platelet transfusion within 90 minutes of brain imaging. Most patients were taking aspirin and if randomized to platelets received one platelet concentrate (5 donor units).  Those taking ADP receptor inhibitors such as clopidogrel (Plavix) and randomized to platelets received two platelet concentrates.  Primary outcome was a shift towards death or dependence rated on the modified Rankin Scale at 3 months.

Results:  The odds of death or dependence were significantly higher in the 97 patients in the platelet group as compared with the 93 patients in the standard care group (odds ratio 2.05, 95% CI 1.18-3.56). Serious adverse events were more frequent in the platelet group (42% vs. 29%) including significantly increased rates of serious adverse events due to ICH, and hospital mortality was higher in the platelet group as well (24% vs. 17%). 

Limitations:  an unknown number of patients were eligible for the trial, and the small trial size with limited enrollment over 6 years are threats to the external validity of the study.  There were also a number of patients enrolled who met an exclusion criteria, or protocol violation.  This trial specifically evaluated patients not heading to the OR; future study would be necessary to evaluate outcomes in surgical patients with ICH.  This study did not assess platelet function.

Bottom line: mortality and morbidity were higher in patients receiving platelet transfusions after non-traumatic ICH than in patients receiving standard care.   This may be due to potential inflammatory and pro-thrombotic effects of platelet transfusion.

3.      Patanwala AE, et alSuccinylcholine Is Associated with Increased Mortality When Used for Rapid Sequence Intubation of Severely Brain Injured Patients in the Emergency Department.  Pharmacotherapy  2016;36(1):57-63.

A youtube video should be imminent, as of course this article interpretation was performed using LARP (Live Action Role Playing).  For a less dramatic synopsis, this was a chart review from a single academic center.  A total of adult 233 patients with TBI requiring intubation were either intubated with succinylcholine (149 patients) or rocuronium (84 patients).  Agent choice was by provider preference.  Mortality was 23% in both groups, with the same first-pass success rate.  Groups were similar except for a higher rate of hypotension in the succinylcholine group.

 When stratified by “Head Abbreviated Injury Score,” rocuronium mortality was 22% in the low score group and 23% in the high score group, while succinylcholine mortality was 14% and 44%.  Authors therefore attempt to draw the conclusion that in more severely brain injured patients, rocuronium is the preferred neuromuscular blocker.  The only problem with this conclusion is that overall mortality rates for the 2 agents were identical, implying that for the lesser brain injured patients, succinylcholine should therefore be the safer agent.  This indeed was the trend, although it didn’t reach statistical significance.  Especially in this retrospective trial, sub-group analysis by Head AIS should be hypothesis generating. 

Rocuronium needs to be given at 1.5 mg/kg IV to provide similar intubating conditions to succinylcholine, and if dosed correctly, provides several potential advantages (avoid theoretical increase in ICP and hyperK with succinylcholine, longer paralysis facilitates multiple imaging studies, etc).  What’s the downside to rocuronium?  Medical staff forget to initiate a long active sedative immediately after intubating, leading the horrible potential of awake but paralyzed patients.  In addition, for certain clinical conditions such as seizures, a shorter acting paralytic allows for earlier repeat neurologic exams. 

Bottom line, for the stated primary outcome of in-hospital mortality, the 2 agents performed equally well.  The “head AIS” is not meant as an initial injury scoring scale, and GCS, which can be determined at presentation quickly, did not discriminate between rocuronium and succinylcholine.  If only for simplicity and to decrease cognitive load, rocuronium should usually be your neuromuscular blocking agent of choice.

 

Journal Club bottom lines:

1.      In patients with ICH and hypertension, lower SBP to 140s.  Maintaining smooth BP control is more beneficial than an aggressively low target SBP in the 120s.

2.      Platelet transfusions in patients with non-traumatic ICH who are taking anti-platelet agents is associated with increased rates of death or dependence. 

3.      Use rocuronium at 1.5 mg/kg IV instead of succinylcholine.  Except maybe in seizure patients who need early neuro-rechecks.  Just remember to start a long acting sedative right away!!

 

 

Journal Club Potpourri

Journal Club Summary, May 2016: Revisiting antibiotics for abscess; Reconsidering the diagnostic evaluation of Acute Heart Failure; the Power of the probe in diagnosing undifferentiated hypotension.

Many thanks to Mike and Karen Lambert for hosting on a beautiful evening, and to Matt DeStefani, Frank Lee, Mike Stanek, Mike Kennedy, Liz Regan and Bristol Schmitz for their cogent, concise article analyses.

Article 1: Talan DA, et al: Trimethoprim-Sulfamethoxazole versus Placebo for Uncomplicated Skin Abscess. N Engl J Med 2016;374:823–832.

Brief background; CA-MRSA abscess is the culprit behind the huge spike in SSTIs seen in our emergency departments over the past 20 years. While historically abscesses have been successfully treated with I/D alone, CA-MRSA is virulent and pesky, and small studies have suggested a role for antibiotics in addition to I/D, both for improved resolution of infection and to help prevent abscess recurrence. These studies have been underpowered, as the I/D success rate even in the age of CA-MRSA is around 80%, making it difficult to demonstrate small differences in cure rates in small studies. The most recent 2015 IDSA guidelines on SSTI recommend I/D and no antibiotics for simple, uncomplicated abscess.

So, a shout out to Dave Talan and Greg Moran, UCLA EM, for publishing this double blind RCT of 1265 patients &gt;12 yo who presented with uncomplicated abscess to 5 US emergency departments and who were treated as outpatients. While immunosuppression was an exclusion criteria, 11% of included patients did have diabetes and 8% had prior MRSA. About half of wound cultures grew MRSA. Average abscess size was 2.5 x 2 cm, with an average area of erythema of 7 x 5 cm.

The primary endpoint was clinical cure of abscess 7-14 days after end of treatment with either 7 days of 2 DS Bactrim bid or placebo. Clinically important secondary outcomes included the need for surgical drainage, hospitalization, development of new infection in patient or household contacts, and invasive infections.

Results: Clinical cure was 80.5% in the Bactrim group, 73.6% in the placebo group; an ARR 7% for cure with a NNT of 14. There was still a high cure rate even without antibiotics...likely even higher assuming those lost to follow-up were clinically cured. Bactrim was superior to placebo for most secondary outcomes, including a 7% ARR in skin infections at new sites (10.3% vs. 3.1%). Subsequent invasive infections were rare in both groups. Adverse events were mild and most commonly GI (43% in Bactrim group vs 36% in placebo group). No C. difficile was reported.

So why not just give everyone with an abscess Bactrim after the I/D? Well, for starters, the majority of abscesses still resolved without antibiotics. Somehow, they didn’t see any, but you have to consider C. difficile. Also cost, the nuisance of non-specific GI side effects, and the potentiation of other medications, especially drug/drug interactions for Bactrim with Coumadin and oral hypoglycemics. They also didn’t see Stevens Johnson Syndrome, which is a rare but potentially life threatening complication of Bactrim. Antibiotic resistance is always a concern.

Maybe this is a time for shared decision making? Pretty simple talking points; 7% ARR for cure or recurrence, but baseline at least 74% cure. They didn’t look at subgroups...maybe push more for antibiotics in a patient with DM or with a large area of cellulitis? Could consider a “wait and see” prescription, although treatment failures may need additional I/D and should probably be re-seen.

NB, if prescribing antibiotics, remember they used 2 Bactrim DS bid for 7 days. Current IDSA guidelines say either 1 or 2 DS tabs bid is acceptable.

Show of hands? A minority of people in the room would take the Bactrim if they had an uncomplicated abscess. Interestingly, nobody in the room admitted to ever having a MRSA abscess....hmmmm.

Article 2: Martindale JL, et al: Diagnosing Acute Heart Failure in the Emergency Department: A Systematic Review and Meta-analysis. Acad Emerg Med 2016;23(3):223-42

It’s an age old question...how to accurately diagnose acute heart failure?

The ED diagnosis of AHF based on history, exam, CXR and ECG is discordant with the final discharge diagnosis in nearly one out of four cases.

In this review of 57 studies with 18,000 patients, 46% of whom were diagnosed with AHF, the authors examined the test characteristics of history and physical exam findings, ECG, CXR, BNP, Ultrasound, Echo, and Bioimpedence devices in patients presenting to the ED with dyspnea.

Not surprisingly, history and exam don’t do so well. There are many overlapping historical and exam features in our patients who often have multiple co-morbidities causing dyspnea. Unfortunately, authors also looked at each feature independently, instead of considering the power of gestalt of multiple history/exam variable. In any case, S3 has the highest LR (+) of 4, but this is found in only 13% of patients with AHF.

ECG: not helpful.

CXR: pulmonary edema on CXR has an LR (+) of 4.8, but CXR has a known poor sensitivity for AHF and a normal CXR does not help rule out CHF.

Echo: if reduced EF, echo can be helpful, except it doesn’t take into account 50% of CHF patients with preserved EF...also may be able to get the EF from recent echo/EMR, so may not add new information. Reduced EF as determined by visual estimation had a LR (+) of 4.1.

BNP was useful as a rule out, with a LR (-) of 0.11 when BNP was &lt; 100 pg/ml. BNP had a LR (+) of 7 when BNP was &gt; 1000 pg/ml, but values in between had unreliable LRs. Point made in the article that dichotomizing a continuous variable such as BNP is not the optimal use of the test. Also, most of the studies evaluating BNP excluded patients with renal failure, leading to fewer false-positives and inflating the test’s specificity.

There have been limited studies of Bioimpedence devices, with heterogeneity in the data analysis, but some initial promising results with segmental bioelectrical impedence analysis.

The overall winner....Lung Ultrasound with a LR (+) of 7.4 when B lines were visualized in multiple lung zones (hint, Mike Lambert likes using the windows at the medial costosternal articulations) and LR (-) of 0.16 when B-line pattern was absent.

Biggest limitation of this paper (beyond its mind numbing length)? Gold/criterion standard diagnosis of AHF is still clinical and consensus based, incorporating the clinical data and objective test results...leading to potential incorporation bias (study evaluates features that are also used to define the final diagnosis).

Bottom line: Use Ultrasound, to both rule in and rule out AHF. Gestalt history and physical may be helpful, but no individual H/P finding except for the elusive S3 is reliable. Very low or very high BNPs can help. Low EF on echo is useful, but misses 50% of patients with EF preserved AHF.

Article 3: Shokoohi H, et al: Bedside Ultrasound Reduces Diagnostic Uncertainty and Guides Resuscitation in Patients With Undifferentiated Hypotension. Crit Care Med 2015;43:2562-9.

In an ideal world, as the ultrasound probe becomes your sixth digit, and a rapid US protocol for the patient with undifferentiated shock will improve diagnostic accuracy and certainty.

In this single center study, attendings trained in critical care US or US fellows performed ultrasounds (4 view cardiac, 8 view lungs, IVC @ hepatic vein, & FAST/Aorta scan) on 118 adult ED patients with undifferentiated shock defined as SBP &lt; 90 mm Hg after at least 1 liter NS IV. Patients with obvious sources of hypotension, or trauma-related hypotension were excluded. Clinicians caring for patients completed a pre/post US form indicating how the results of the protocolized US performed by an independent physician influenced their diagnostic certainty, diagnostic ability, and treatment/resource utilization. Primary outcome  was the change in the treating physician’s diagnostic certainty pre/post US and the concordance of post-US ED diagnosis with final chart review diagnosis.

There was a 28% decrease in diagnostic uncertainty after US, and an increase in the proportion of patients with a definitive diagnosis from 0.8% to 12.7%. Overall, the leading diagnosis after US demonstrated excellent concordance with the final diagnosis. A quarter of patients had significant changes in management after US, and significant changes in diagnostic imaging, consultation, and ED disposition. A subgroup of cases was described where ED US identified serious and time sensitive diagnoses (eg PE, ruptured splenic artery aneurysm, AAA).

Limitations: uncertain how much clinical information was available to physician prior to performing the scans, and this would potentially impact the post-test certainty of diagnosis. Likewise, initial resuscitation measures, including average amount of initial fluids given, was not included. How long do these scans take?? Lambert thinks maybe 10 minutes if you do it a bunch. At the same time, for many patients, after a targeted history, the complete protocol is probably not necessary, and scan time therefore decreases. This study didn’t evaluate the impact on clinical, patient oriented outcomes, including the impact of US on morbidity, mortality, ICU stay, etc. Finally, pre-US protocol diagnosis accuracy and post-US diagnosis accuracy were not directly compared, again likely secondary to lack of standardization of initial clinical information and initial resuscitation measures.

Bottom line: Kerwin-make US a habit. Barounis-it will make you a faster clinician. Team US in general: this is an important subgroup of patients, with potentially time sensitive diagnoses. US: do more, be better, save lives. 

Influenza and oseltamivir

Journal Club Synopsis, January 2016.  Oseltamivir-Prescriptions and Profits.

Many thanks to Branka Milicev for tasty Italian food and beautiful views from her south loop penthouse!

The Tamiflu controversy heated up in 2014.  Turns out Roche, manufacturer of oseltamivir, had suppressed a number of unpublished trials.  A group of researchers went after Roche for several years, and after much public shaming, Roche finalized released these data.  Studies incorporating the unpublished data were published in the April 9, 2014 issue of BMJ as well as our included Family Practice article.  The authors of the BMJ articles also updated the pertinent Cochrane review in 2015, which covers the use of all neuraminidase inhibitors in preventing and treating influenza.

Article 1: (Regan, Navarette)

Ebell MH, et al. Effectiveness of oseltamivir in adults: a meta-analysis of published and unpublished clinical trials.  Family Practice 2013; 30:125–133 doi:10.1093/fampra/cms059

This meta-analysis includes published and unpublished double-blinded, placebo-controlled RCTs of oseltamivir in adults with suspected or confirmed influenza that reported the primary outcomes of duration of symptoms, complications, or hospitalizations.  All of these studies were limited to patients presenting within 36 hours of symptom onset.  Pregnant patients were excluded by most of the reporting articles.  Authors were granted access to trial registries maintained by the drug manufacturer.  Eleven studies (3 published, 8 unpublished) with a total of 4769 patients met inclusion criteria. 

For the intention to treat (ITT) population, the mean reduction in duration of symptoms was 21 hours.  There was no significant reduction of symptom duration in the elderly and in patients with chronic disease, two populations felt to be at higher risk.  One study stratified by time of symptoms:  reduction of symptoms by 29 hours if oseltamivir was given within 24 hours of symptom onset, 15 hour reduction if given between 24-36 hours of symptom onset.

Overall, there was no difference in the likelihood of hospitalization and no reduction in the rate of infectious complications requiring an antibiotic when acute bronchitis was excluded.  Authors excluded acute bronchitis as current guidelines do not support the use of antibiotics for acute bronchitis.  The risk of pneumonia was reduced in the population with PCR confirmed influenza infection (NNT = 111), but not in the general ITT population.  There was only one death reported, in the placebo group. 

Downsides to Tamiflu?  Cost is about 100$.  Side effects of oseltamivir were not specified in this meta-analysis, but other reports state rates of nausea and vomiting between 5-10%.  There are less common neuropsychiatric and renal complications as well as headache.  An additional concern is the expected increase in drug resistance with liberal oseltamivir use.  Authors also comment on the questionable motivations of the manufacturer not publishing multiple studies, and the break in the public trust when patients are recruited and enrolled in studies whose results are never used for the intended purpose of advancing medical care.

Limitations of the meta-analysis include the limited reporting of complications in the ITT population.  Existing literature includes only small numbers of hospitalizations and serious complications, which limits comparisons, especially for high-risk patients.

From our presenters: they are less likely to prescribe Tamiflu.   One more day of flu has different meanings to different people, depending on their income, work situation, future exposures, comorbidities.   

Bottom line:  In this large meta-analysis of published and unpublished studies, there is no evidence that oseltamivir reduces the likelihood of hospitalization or acute infectious complications.  There is a small reduction in the risk of pneumoniabut this was only seen in lab confirmed influenza (NNT =111).  There is a reduction of about one day in duration of symptoms if oseltamivir is given within the first 36 hours of illness, but treatment is associated with side effects of nausea and vomiting.

 

Article 2: (Einstein, Ede)

Muthuri SG et al. Eff ectiveness of neuraminidase inhibitors in reducing mortality in patients admitted to hospital with influenza A H1N1pdm09 virus infection: a meta-analysis of individual participant data. Lancet Respir Med 2014;2: 395–404.

So, if oseltamivir doesn’t help prevent infectious complications or hospitalizations in outpatients, does it at least help in sicker patients with flu who require hospitalization? 

This is a meta-analysis of retrospective, observational data from the 2009-2010 Influenza A H1N1 pandemic.  It included data for 29,234 patients from 78 studies worldwide, who were admitted to hospital with clinical or lab confirmed influenza A H1N1.  Primary outcome was the association between neuraminidase inhibitor treatment and mortality, adjusted for treatment propensity and confounders.  Although the authors state in their methods that they attempted to include both observational studies and RCTs, in their conclusions they refer to using retrospective, observational data and in the appendix, there is no further clarity regarding the methodology of studies included. 

In adults, neuraminidase inhibitor treatment was associated with mortality risk reduction, adjusted odds ratio 0.81; 95% CI 0.70-0.93.  Treatment within 2 days of symptom onset vs. late treatment, treatment within 2 days vs. no treatment, and treatment of pregnant women and critically ill adult patients were all associated with even greater mortality risk reductions.  If treatment was started within 2 days, NNT (to save a life of an admitted pt) was 31.

Combining all subgroups, there was no mortality benefit with treatment delayed more than 2 days after symptom onset; however there was still benefit of delayed treatment in patients admitted to ICU.  The association with reduced mortality was not statistically significant in pediatric patients. 

What are the problems with this study?

First, funding was by Hoffman –La Roche, makers of oseltamivir, and the main author was a paid consultant for Pharma and former employee of Roche.  As seen from the first article, there have been big issues with suppressed data and overstatement of treatment effect with this drug.

The data were retrospective and observational, implying potential for lots of residual confounders and missing information.  Although 401 centers were contacted to contribute, only 78 centers were included.  Finally, the quality of the included studies has been described as “medium to poor,” with “clear evidence of unadjusted biases.” (theNNT.com).

Bottom line:  In this meta-analysis with significant limitations, oseltamivir treatment within 2 days of symptoms was associated with a mortality reduction in hospitalized adults, with the greatest benefit seen in pregnant women and critically ill patients.

 

Article 3: (Hart, West)

Lenzer J. Why aren’t the US Centers for Disease Control and Food and Drug Administration speaking with one voice on flu? BMJ 2015;350:h658 doi: 10.1136/bmj.h658

Ok, I’ll get a little political.

From “Tamiflu, a Nice Little Earner:-BMJ, 2014.

Since its launch in 1999, oseltamivir has generated cumulative sales in excess of $18 billion for Roche. Half of the total expenditure was by governments and companies around the world for stockpiles for pandemic preparations. The US alone spent more than $1.3 billion buying a strategic reserve of antivirals.  Most have never been used, and today the US stockpile consists of more than 65 million treatments. 

Back to the Lenzer’s editorial:

The FDA disagrees with the CDC  “Take 3” campaign that claims oseltamivir “saves lives.”  The FDA has stated that oseltamivir “has not been proven to have a positive impact on the potential consequences (such as hospitalizations, mortality, or economic impact) of seasonal, avian, or pandemic influenza.”

CDC director Thomas Friden has made expansive claims about the benefits of neuraminidase inhibitors for non-admitted patients, including the implication of a general mortality benefit.  This approach has been called irresponsible, and an appeal of emotion instead of science.  Oseltamivir costs around 100$, which for many patients is a substantial cost for a marginal benefit.  Sending large numbers of otherwise healthy people to the ED who have influenza but who won’t actually benefit from oseltamivir may actually increase the spread of the virus around crowded ED waiting rooms.

The “CDC Foundation” provides funding to the CDC.  The CDC received a directed donation of $198,000 from Roche (maker of oseltamivir) via the Foundation for the “Take 3” flu campaign.  Step 3 of “Take 3” encourages people to “take flu antiviral drugs if your doctor prescribes them.”  Overall, the CDC receives millions of dollars annually from Big Pharma, channeled through the CDC Foundation.  The reputation of the CDC as being free of industry bias is viewed by some as now compromised.

The CDC cites the Lancet’s 2015 Dobson meta-analysis as demonstrating a reduction in hospitalization after outpatient treatment with oseltamivir.  This study was indirectly sponsored by Roche, and according to Jeanne Lenzer, “all four researchers received industry funding either directly or through industry donations to organizations that directly funded the study (so-called “pass-through” money).”  The Dobson paper’s conclusions are far different than the conclusions from Journal Club’s meta-analysis by Ebell and the 2014 BMJ systematic review by Jefferson, both of which included previously unpublished data from Roche.

Even after the 2014 articles in the BMJ and Family Physician, the CDC’s recommendations regarding neuraminidase treatment for influenza are unchanged.

Really though, if you want the dramatic interpretation, please tune in to youtube and our very own thespians Elise Heeringa and Nathan West.  This was forwarded to Jeanne Lenzer at the BMJ (author of the editorial), who was very impressed J

https://www.youtube.com/watch?v=WpzMS3c2090&feature=youtu.be


y Journal Club bottom line:  for outpatients, there’s just no data to support treatment with neuraminidase treatment to prevent serious complications, hospitalization, or death.  At best, for outpatients, if treated within 36 hours of symptom onset, you’ll get 21 hrs of decreased illness, for a 100$ drug with up to 10% incidence of N/V.  For adult inpatients, if oseltamivir is started within 2 days, there is likely a mortality benefit.  For admitted kids, overall data says mortality same with/without oseltamivir.  However, there is a December 2013 Pediatrics study (Louie et al) of kids admitted to the ICU that demonstrated a mortality benefit. 

Point of Care Ultrasound

Thanks to Sheila and Ron Bonaguro for the early Thanksgiving feast!!  Outstanding presentations by Ari, Patrick, Natalie, Kennedy and Stephen. 

 

Pushing the depth of field for point of care ultrasound?  Use of US to predict futility of resuscitation after medical and traumatic cardiac arrest, and as a potential alternative to the trauma pan-scan CT.


Article 1:  Flato UA, et al:  Echocardiography for prognostication during the resuscitation of intensive care unit patients with non-shockable rhythm cardiac arrest. Resuscitation  2015  Jul; 92:1-6. 

This was a Brazilian prospective, observational cohort study of 49 ICU patients with in hospital asystole or PEA cardiac arrest. Of 88 eligible patients, 39 were excluded. PEA without contractility was classified as electromechanical dissociation (EMD), and PEA with contractility as pseudo-EMD.  Two echo trained intensivists performed transthoracic echos. Rates of ROSC were 70% for pseudo-EMD, 20% EMD, and 24% for asystole.  Good ROSC percentages for all groups, but survival to hospital discharge was only seen in pseudo-EMD patients, and discharge is what counts.  Four patients survived to 180 days, all in the pseudo-EMD group, with CPC of 1, 1, 1, 2.  Echo was feasible, with maximum duration of 10 seconds, so non-disruptive to the ongoing code.  Echo also helped identify underlying etiology of arrest in selected patients, eg unexpected tamponade.  This was a very small study, and conducted in an ICU with a large number of DNR patients who were never entered into study, so different from our ED population.   

This study reinforces our usual practice of using echo to verify presence/absence of cardiac contractility and guide futility of resuscitation.  Blaivas and Fox (go ACMC!!) published a larger study in 2001 demonstrating 100% mortality in patients with asystole or PEA cardiac arrest and no cardiac contractility on bedside echo. This modality helps conserve resources (time and personnel), and may identify the underlying reason for code.  In the future, in young otherwise healthy patients with arrest, an echo demonstrating contractility may help risk stratify for ECMO.

Bottom Line:  Rather than pulse check, consider echo as the more reliable marker for viability.  Also remember end tidal CO2 to guide prognostication.  

(AHA 2015 ACLS guidelines:  “In intubated patients, failure to achieve an ETCO2 of greater than 10 mm Hg by waveform capnography after 20 minutes of CPR may be considered as one component of a multimodal approach to decide when to end resuscitative efforts but should not be used in isolation.”)


Article 2:  Inaba K et al:  FAST ultrasound examination as a predictor of outcomes after resuscitative thoracotomy: a prospective evaluation. Ann Surg  2015  Sept;262(3):512-8. 

Prospective study between 10/2010 to 5/2014 of 187 trauma patients undergoing resuscitative thoracotomy  (RT) at LAC-USC, and who received FAST before or concurrent with RT.  Thirty-six other patients (16%) received RT but no FAST; these all died.  Median transport time was 33 minutes.  Mechanism was half blunt, half penetrating.  Half lost vital signs at the scene.  FAST was adequate if able to determine presence/absence of cardiac contractility and pericardial fluid.  Seven FAST exams (4%) were inadequate.   Sustained cardiac activity was regained in 48%, but overall survival was only 3.2%, with an additional 1.6% proceeding to organ donation.  Unclear how long FAST took to perform.  Primary outcome measure was survival to discharge or organ donation.  Cardiac motion on FAST was 100% sensitive and 74% specific for the identification of survivors to discharge and organ donors.  The likelihood of survival if pericardial fluid and cardiac motion were both absent was zero. The thought at JC was that authors were trying to stress the importance of pericardial fluid being associated with potentially higher survival, but in reality, the bright line was presence or absence of cardiac contractility:  no contractility =  no survival.  Not entirely similar to our population, as at USC-LAC stab wounds to the box go immediately to the OR, regardless of initial blood pressure, and therefore these patients were not included in the study.  Additionally, we rarely perform RT on blunt trauma.  Interestingly, 4 blunt trauma patients “survived” (did not specify discharge or donor). Four of the six discharged patients were cognitively intact. 

Authors conclude that FAST represents an effective method of determining those patients who do not warrant the risk/resource burden of RT from those who may survive.  It’s a modality to tell you when to stop (NPV), not a modality to tell you when to start (PPV).

Bottom line:  In this study, if FAST had been used to select patients for resuscitative thoracotomy, all survivors would have been identified and RT rate would have been more than cut in half. 


Article 3.  Dehganzada ZA et al:  Complete ultrasonography of trauma in screening blunt abdominal trauma patients is equivalent to computed tomographic scanning while reducing radiation exposure and cost. J Trauma Acute Care Surg 2015  Aug;79(2):199-205. 

It’s tempting, right? No radiation, portable, quick, relatively inexpensive, and repeatable....there have been numerous efforts to reduce CTs by using physical examination algorithms + US in blunt torso trauma. But as a 2015 Cochrane review discussed, the overall sensitivity for FAST in blunt trauma is low. Trying to incorporate observation periods is challenging too-it’s easy for a busy trauma service to be overwhelmed.   

This study really released the fury of Stephen Jamieson.  Statistical back-fists, jabs at the inconsistent definitions, and an eventual throw-down of all the conclusions-a sight to behold.

In this 11 year retrospective single center study of 19,128 blunt abdominal trauma patients, 66% initially underwent ultrasound, and 34% CT scan.  A “Complete US of Trauma” (CUST) included 7 abdominal regions:  CUST does not equal FAST.  In the CUST group, initially 4% were “positive”, meaning injury confirmed by exploratory laparotomy or CT.  In the CT group, 7.6% positive.  The study defined false negative US as requiring a laparotomy, except when it defined false negative as having an abnormal CT.   Confusing and suspect. Overnight patients received CT.  However, there were also patients who received CT rather than CUST at surgeon discretion regardless of time of day (especially if CUST was poor quality, patient obese, seat belt injury, if hematuria, if significant abdominal pain without operative indications, or if spinal and/or pelvic fractures were suspected-hmmm, higher risk patients shuttled to CT rather than US?  This will change the test characteristics!)  Sensitivity = TP/TP + FN, or for CUST 199/199 + 86 = 70% using abnormal CT following normal US as the definition of FN.  

Mortality was higher in the U/S group (1.8% vs. 1.2%, p = 0.03, NN2Kill=166).  Mortality differences became insignificant when results were conveniently adjusted for age > 65 years or head injury.  Not clear why they felt this was justified, especially as CT group was sicker and with a higher percentage of head injuries-would expect CT group to have higher mortality.

The authors acknowledged that US is lower sensitivity/higher specificity, but then tried to justify US as a rule-out test.  As Kennedy stressed, it should instead be used as a rule-in test:  useful only if abnormal.

UCSD was averaging 4 trauma activations/day (we get about 10), with overall relatively low injury severity scores and <2% laparotomy rate.  As Harwood said, there may be a low risk population where US would be useful as a screening exam, but this study doesn’t help us figure it out.   Conclusions touting cost and radiation savings are also suspect:  no idea if CT is being used appropriately, and as UCSD now has dedicated ultrasonographers for this protocol 24/7, difficult to assess true costs.

Bottom line:  CT is still the winning modality for serious blunt abdominal trauma.  And don’t mess with Jamo.

O2 in MI; O2 Delivery in Hypoxemic Respiratory Failure; ROSC and the Cath Lab.

Journal Club Synopsis September, 2015:  O2 in MI; O2 Delivery in Hypoxemic Respiratory Failure; ROSC and the Cath Lab.

Many thanks to Mike Marynowski for hosting (skyline views and an outdoor big screenTV-what??).  Outstanding synopses by Braden, Elise, Franklin, Dr. Ede, Rachel and Dr. Burns.

Article 1:  We should think of oxygen as a drug, with benefits and harms. There has been suggestion since the 1970s that O2 may be harmful in ACS, through the reduction of coronary blood flow,  increase in coronary vascular resistance, and generation of reactive oxygen species. The latest AHA STEMI guidelines give no clear recommendation for O2 therapy in normoxic patients.

Stub D et al. Air Versus Oxygen in ST-Segment-Elevation Myocardial Infarction. Circulation  2015;131(24):2143-50.   AVOID Trial.

Editorial:  Nedeljkovic ZS, Jacobs AK:  Oxygen for ST-Segment–Elevation Myocardial Infarction Still Up in the Air.  Circulation  2015;131:2101-3.

In this multicenter, prospective, RCT from Australia, 441 normoxic patients with STEMI received O2 (8 L/min by facemask) or no O2, pre-hospital until admission to cardiac care ward.  Primary outcome was MI size as assessed by degree of cardiac enzyme elevation-troponin and CPK.  Secondary outcomes included recurrent MI, arrhythmia, and MI size assessed by cardiac MRI at 6 months.  There was no significant difference in troponin elevation between the two groups, although there was a significant increase in mean peak CPK in the O2 group compared to the no O2 group.  Rate of recurrent MI was significantly higher in the O2 group (5.5% vs. 0.9%, p=0.006).  The O2 group also had a trend towards higher frequency of cardiac arrhythmias (40% vs. 31%) and significantly increased MI size on cardiac MRI.  The kicker was mortality.  Six-month mortality was 3.8 with O2 vs. 5.9 with air (NNtoKill=48).  This was non-significant statistically, but a concerning trend, and engendered significant discussion at JC, especially as the study’s primary outcome was not clinical/patient oriented.  Troponin/CPK elevation are disease oriented outcomes, and very rough surrogates for outcome and prognosis.  Unfortunately, the trial was not powered for clinical outcomes.  Authors are to be congratulated for completing a challenging study involving delayed consent initiated in the field, but statistical methods were controversial, involving “imputation”, or presumption of results when data were missing (statistics on made up data).  The amount of O2 provided was also questioned, as nasal cannula rather than mask is more the norm in the USA.  The accompanying editorial states that the routine use of oxygen for patients with STEMI remains “up in the air”, and the concerning mortality trend led the majority of folks at JC to vote to continue providing routine O2 to this patient population.  Stay tuned:  a large Swedish Registry randomized trial examining this question is currently enrolling patients, and is powered for both morbidity and mortality.

 

Article 2:  We all love us some BIPAP in respiratory failure due to COPD or CHF, as noninvasive positive-pressure ventilation reduces intubation and mortality rates in these patients.  However, in patients with non-hypercapnic acute hypoxemic respiratory failure (think really bad pneumonia), noninvasive ventilation is associated with high failure rates and particularly high mortality.  Maybe not such a great idea to always reach for the Darth Vader mask...what’s the optimal O2 delivery in acute hypoxemic respiratory failure?

Frat JP, et al: High-flow oxygen through nasal cannula in acute hypoxemic respiratory failure.  N Engl J Med  2015;372:2185-2196.

In this multicenter, open label RCT trial of 310 ICU pts with non-hypercapnic acute hypoxemic respiratory failure, which excluded patients with CHF, asthma, and COPD, patients received 1 of 3 treatments: high flow O2 via nasal cannula (HFNC), noninvasive positive-pressure ventilation, or standard oxygen therapy via face mask.  Respiratory failure criteria were defined, as were criteria for intubation.  Primary outcome was proportion of patients intubated at 28 days; secondary outcomes included all-cause ICU and 90 mortality rates.  Twenty-eight day intubation rates were 38% vs. 50% vs. 47%, respectively; a non-significant trend favoring HFNC.  Ninety-day all-cause mortality was significantly lower for HFNC (12% vs. 28% vs. 23%, NNT=6 or 9).  ICU complication rates did not significantly differ between the three groups.

This study supports the superiority of HFNC in acute hypoxemic respiratory failure, compared with noninvasive ventilation or O2 by face mask.  Noninvasive ventilation performed poorly.  Caveats?  This was an ICU, rather than an ED study.  We don’t see many patients with respiratory failure from pure pneumonia.  However, for this patient cohort, consider high-flow O2 via NC instead of BIPAP.  Yes, we have it in the ED.  Need to dial in flow rate (max 60 liters/minute, this study used 50 L/min), and FiO2.  HFNC is comfortable, well tolerated, and allows patient to eat, drink, and communicate. 

Article 3:  You have ROSC after cardiac arrest!  Patient is hemodynamically optimized and cooling, O2 sat is 94%, and the ECG shows.....not a STEMI L.    2010 AHA guidelines state: “consideration of emergent coronary angiography may be reasonable even in the absence of STEMI” in post-arrest patients.  Should you push for cardiology to take them to the cath lab?

Kern KB, et al: Outcomes of Comatose Cardiac Arrest Survivors With and Without ST-Segment Elevation Myocardial Infarction: Importance of Coronary Angiography.  JACC Cardiovasc Interv  2015 Jul;8(8):1031-40. 

In this retrospective review of 746 comatose post-cardiac arrest patients including 198 with STEMI (26.5%) and 548 without STEMI (73.5%), overall survival was greater in those with STEMI compared with those without (55% vs. 41%; p = 0.001). However, in all patients who underwent immediate coronary angiography, survival was similar between those with and without STEMI (55% vs. 58%; p = 0.60). A culprit vessel was more frequently identified in those with STEMI, but also in one-third of patients without STEMI. The majority of culprit vessels were occluded (STEMI, 93%; no STEMI, 69%).  An occluded culprit vessel was found in 74% of STEMI patients and in 23% of non-STEMI patients. Among cardiac arrest survivors discharged from the hospital who had presented without STEMI, coronary angiography was associated with better functional outcome (93% vs. 79%; p < 0.003, NNT=7).

Basically, this study attempts to show that early coronary angiography is associated with improved outcomes, regardless if STEMI is present on ECG.  An occluded culprit vessel was found in 23% of non-STEMI patients who went to cath lab.  This is aligned with prior studies demonstrating up to 50% of patients with ROSC after cardiac arrest having an acutely occluded coronary vessel, not well predicted by the presence of STEMI on ECG (Spaulding, 1997, NEJM).

So what’s the problem?  First, this was a study of 6 hospitals, with widely variable cath lab protocols, and additional individual cardiologist variability.  Coronary angiography was only performed in 45% of non-STEMI patients.   The non-STEMI patients who went to cath lab are likely very different from those who didn’t-the fatal flaw of this study.  It’s also a retrospective registry trial, therefore lots of other limitations.  A quarter of the non-STEMI patients who went to cath lab went in a delayed fashion, which has been shown to be of no significant benefit in stable, asymptomatic patients (OAT trial NEJM 2006; 2013 AHA STEMI guidelines).  The AHA STEMI guidelines specifically classify PCI of a non-infarct artery at the time of PCI for STEMI as “Class III-harm”; in other words, PCI should be not be driven by an interventionalist’s oculo-occluded reflex.  If the occluded vessels identified in the 23% of the non-STEMI patients are truly culprit, then positive outcomes may be expected, but the study’s conclusions are not supported by its methodology.  How to improve?  Publish the data for hospitals who take all ROSC patients (STEMI and non-STEMI) to cath lab.

 

 

 

 

 

 

 

CV Potpourri- Low risk chest pain/CHF treatment

Journal Club Synopsis:  ED treatment of CHF.  Early Discharge of Low Risk Chest Pain

Thanks to Andrea and Keith Carlson for hosting, especially for the wicked ping-pong!

Outstanding critiques of the articles by Nathan West, John Paik, Mike Kennedy, Aris Alexander, Katie Burns and Jessica Ede.

1.  Collins, SP, et al:  Early Management of Patients With Acute Heart Failure: State of the Art and Future Directions—A Consensus Document from the SAEM/HFSA Acute Heart Failure Working Group.  Acad Emerg Med 2015; 22:94–112.

Heart failure results in 1 million ED visits annually.  Based on guidelines from national and international cardiology societies, this consensus document from SAEM and the Heart Failure Society of America discusses best practices for the diagnosis, treatment, and disposition of ED patients with acute heart failure (AHF). 

Unfortunately, literature suggests that there is no historical or exam finding with both high sensitivity and high specificity for the diagnosis of AHF.  Prior HF is the most useful historical feature, but the differential diagnosis of dyspnea remains broad even in these patients.  The elusive S3 gallop has the highest LR+, with modest LR+ for the historical features of PND, orthopnea, and peripheral edema, and for the exam findings of HJR and JVD.  Up to 20% of patients with AHF will have no congestion on their ED CXR, especially in late-stage heart failure.  ECG may suggest underlying precipitants for AHF, and A fib has a high LR+ for AHF.  BNP testing is strongly recommended, although levels can be affected by age, gender, weight and renal function, and can be elevated in other conditions causing myocardial stretch such as ACS and PE.  Patients with HF and preserved LVEF have proportionally lower BNP levels.  Ultrasound can be extremely useful, both to identify AHF (better test characteristics than CXR!), and to use IVC collapse index to evaluate volume status. 

Currently, treatment recommendations vary between societies, reflecting in part the need for high quality ED clinical trials with clinically meaningful outcomes.  Using an algorithm to classify patients, both by initial blood pressure and an assessment of volume status, can help to more effectively tailor treatment to the individual patient.  IV loop diuretics are used frequently, but should be started at relatively low doses, e.g. the IV equivalent of the patient’s usual oral daily dose (DOSE trial).  Although still included in some guidelines, nesiritide is not useful, and low dose dopamine has also not been shown to be helpful (ROSE trial).  Especially in patients presenting with elevated BP, first line treatment should include high dose IV nitrates, as these patients usually have a volume distribution more than volume overload problem.  Early non-invasive ventilation in severe AHF is commonly used and associated with improved symptoms and lower rates of intubation.

Morphine is mentioned as an anxiolytic, however benzodiazepines are more generally ordered for this indication.  Starting inotropes and management decisions about patients in cardiogenic shock will usually be made after early consultation with cardiology.  AHF requiring hospitalization is recognized as a fundamental change in the clinical trajectory of a patient with heart failure.  Our hospital treatment does not lead to clear improvement in survival or re-hospitalization rates. 

Currently 80% of patients with AHF are admitted.  Features including renal dysfunction, low BP, hyponatremia, and elevated troponin or BNP have been associated with increased morbidity and mortality.  However, the absence of high risk features does not equate to the patient being low risk and safe for discharge.  While several decision instruments for early discharge of patients with AHF have been derived, there is no accepted, well-validated published tool.  Our ACMC CHF Observation Protocol is an option in selected patients.  Early follow-up and a stable social situation are key factors to the safe implementation of observation protocols.

 

2.  Mahler S, et al: Performance of the 2-hour Accelerated Diagnostic Protocol Within the American College of Radiology Imaging Network PA 4005 Cohort. Acad Emerg Med  2015;22:452-60.

Bottom line:  This trial of 1,140 ED patients being evaluated for ACS failed to validate ADAPT as a safe early discharge strategy, however questions were raised about the use of revascularization as a valid endpoint.  Differences in patient populations and health care systems may explain the lower sensitivity of ADAPT in this trial, but further studies are needed.

This was a secondary analysis of 1,369 patients already enrolled in a multi-center CTCA trial in 5 tertiary care US EDs.  Data elements were collected prospectively.  Patients were at least 30 yo, with symptoms suggestive of ACS, and either classified as “low risk” or “at risk” by ADAPT, the 2 hour accelerated diagnostic protocol which includes 0 and 2 hour troponins, ECGs, and a TIMI score.  Eligible patients had initial TIMI scores of 0 to 2, and a non-ischemic ECG on presentation. 

Patients designated low risk and appropriate for early discharge by ADAPT have a TIMI score of 0, ECG with no new ischemic changes, and negative troponin x 2. 

Primary outcome was the rate of MACE (cardiac death, MI, or coronary revascularization) within 30 days of presentation.  Secondary outcomes included the rate of patients identified as safe for early discharge, and abnormal stress or CCTA testing.

Of the 1,140 patients with complete biomarker data, 2.7% had MACE.  ADAPT identified 551 of 1,140 (48%) as low risk and appropriate for early discharge; 5 of these 551 (0.9%, 95% CI=0.3% to 2.1%) had MACE at 30 days.  Of these 5 patients, there were no deaths, one patient with MI, and 5 with revascularizations.  Sensitivity of ADAPT for MACE within 30 days was 84%.  Serial troponins alone were 32% sensitive for MACE. 

These findings conflict with prior very positive studies of the ADAPT strategy. In part this may be due to the inclusion criteria, as the umbrella CCTA trial excluded patients with elevated troponin, whereas prior ADAPT validation studies included patients with elevated troponins who would be identified as “at risk” by ADAPT, thereby boosting the sensitivity of those studies.   In addition, 4/5 of the low risk patients with MACE had revascularization procedures without MI.  The relevance of revascularization as an outcome measure is uncertain.  As discussed at Journal Club, the revascularization rate for patients in the USA is high, and validations of ADAPT in other parts of the world demonstrated much lower percentages of revascularizations in their MACE composite endpoints.  It was also discussed that only 1 of the 5 “missed MACE” patients really appeared to be a true miss.

There were a number of low risk patients with abnormal CCTA or stress testing in this study.  Authors emphasize the importance of early therapy in these patients, reinforcing the need for close follow-up for patients discharged who do not receive higher level diagnostic testing in the ED.

 

3.  Mahler S, et al: The HEART Pathway Randomized Trial: Identifying emergency department patients with acute chest pain for early discharge.  Circulation Cardiovascular Quality and Outcome  2015:8;195-203.

Bottom line:  Use of the HEART Pathway (HEART score + 2 negative troponins) allowed for a higher rate of early discharge from the ED without stress testing compared with usual care in this study of low risk ED chest pain patients.  No patients identified for early discharge had missed MACE at 30 days.

In this study of 282 ED patients with symptoms concerning for ACS, patients were randomized to either the HEART Pathway or Usual Care.  The HEART score includes history, ECG, age, risk factors, and troponin, and a HEART score of 0 to 3 is consistent with low risk.  The HEART Pathway consists of the HEART score + troponins at 0 and 3 hours.  Patients with low risk HEART scores and negative troponins were recommended for discharge without further cardiac testing.  Usual care was based on ACC/AHA guidelines.  The primary outcome was rates of objective cardiac testing (stress or angio) and secondary outcomes included length of stay, early discharge, cardiac related ED revisit, admission, and MACE, all assessed at 30 days. Eligible patients were age at least 21 yo, being evaluated for ACS with ECG and troponin.  Exclusion criteria included STEMI, instability, or additional serious acute medical problems.

Compared with usual care, use of the HEART Pathway significantly decreased objective cardiac testing at 30 days by 12% (69% vs 57%), length of stay by 12 hours, and increased early discharges by 21%.  MACE occurred in 6% of patients overall.

No patients identified for early discharge in either HEART Pathway group or usual care group had MACE at 30 days.  It’s important to note that the primary outcome was objective cardiac testing, rather than a comparison of rates of MACE in the 2 groups, and the study was not powered to detect differences in MACE.  However, the safety of the HEART Pathway has been consistently validated in other trials.

Decision instruments such as the HEART score are useful to support medical decision making, both when having conversations with patients about risk and their disposition preference, and when documenting in the medical record.   Although “gestalt” can be reliable for more senior clinicians, decision instruments in general are particularly useful as an explicit reminder of high-risk features, and help build more reliable heuristics.  

Wellness!!!

NUTRITIONAL BYTES

Articles

How to Eat Healthy Meals At Restaurants...great piece from our friends at the NY Times about how to make smart choices when trying to eat out in the land of the Super Size.

The History of Supersizing...How did we end up with this ridiculous option in the first place?

Recipe

Go out to eat, and try out your newfangled knowledge!

 

WORKOUT OF THE WEEK

Article

Steady-State Cardio Vs. HIIT

 

Workout

30 bicycle crunches

10 pushups

20 airsquats

20 bicycle crunches

5 pushups

20 airsquats

10 bicycle crunches

3 pushups

20 airsquats

 

Repeat 3-5 times

 

 

Wellness Round-Up!

Howdy pardners,

Well, the jig is up. If I wait any longer this is gonna become Biennial Wellness.

In the hopes of betterin' yer corporal existence, its time fer all you liddle doggies ta get along and saddle up fer a rip-roarin', dern-tootin', belly-fillin', butt-kickin' good time.

Giddyup!

 

NUTRITIONAL BYTES

The Government's Bad Diet Advice

The Myth of High Protein Diets

Simple Rules for Healthy Eating

Recipe

Spring Vegetable Stew...this is ridiculously good. Its filling but not heavy, and tastes as fresh as the spring we are supposed to be experiencing. Don't worry if you don't have lard-we just doubled up on the olive oil. Really, any cooking fat will do for this dish. 

 

WORKOUT OF THE WEEK

Christopher McDougall Wants You to Go Outside

The Right Dose of Exercise For A Longer Life

The Early-Morning Workout Flash Mob

Workout

Burpees: 30 seconds

Rest: 30 seconds

Repeat 4-9 times

 

Conference Workout Anyone?

Finally, as many of you are aware, we did a group workout the other day in the middle of conference. Many of you came up to me afterwards and suggested that we do such a workout every conference. As such, I have created a brief (2-question) survey to guage interest. If interest is sufficient, then perhaps we could incorporate this into our conference frequently if not regularly. Thanks for taking the survey and YEE-HAW!!!

Conference Workout Survey

 

 

 

Wellness Topics

Thanks to Dr. John Principe and Dr. Dan Girzadas for hosting the January 2015 Journal Club at Dr. Principe’s WellBeingMD Center.  Appropriately, the topic was wellness, and the delicious and nutritious dinner prepared by the Center’s chef and staff was enjoyed by all.  Outstanding presentations by Drs. Parker, Cirone, Burt, Bamman, Negro and Nejak, as well as the Usastrana demo by Dr. Negro-Namaste.

 

1.     Shanafelt TD, et al: Burnout and Satisfaction With Work-Life Balance Among US Physicians Relative to the 
General US Population.  Arch Intern Med 2012;172:1377-1385.   

 

Burnout, including loss of enthusiasm, cynicism, and low sense of accomplishment, has negative professional and personal consequences.  This survey of US physicians from all medical specialties attempted to evaluate rates of burnout and satisfaction with work-life balance.  Of 27,276 physicians invited to participate, 7288 completed the surveys.  Using the Maslach Burnout Inventory, 46% of all physicians reported at least one symptom of burnout, with Emergency Medicine at the top of the list (>60% of EM physicians), followed by Family Medicine and Internal Medicine.  Compared with a sample of US working adults, physicians were more likely to have symptoms of burnout and be dissatisfied with work/life balance.  Limitations:  there was only a 27% overall response rate, and only 333 emergency physicians.  One always has to question motivation of survey respondents, and wonder about the large group who did not respond.

 

So, how do we avoid/mitigate burnout?  Talk to Cirone, he has it all worked out.  Other voices in the room suggested:  Develop a niche-do something professionally beyond just working your shifts.  Nurture your other passions, outside interests, and relationships.  Retain your sense of humanity; wonder at the amazing intimacy our patients, true strangers, allow us.  You’ll forget more medicine than your patients will ever know-give them a break.  Teach students and residents:  medical, nursing, paramedic.  I didn’t see the movie, but Sam Lam alluded to “Frozen”, and the importance of being pushed to capacity and a little beyond our comfort zone.  Finally, maybe at some point, for some, burnout isn’t bad, but a message that it’s ok to leave, and pursue other interests.

 

2.     Hall KN, et al:  Factors Associated with Career Longevity in Residency-Trained Emergency Physicians.  Ann Emerg Med 1992;21:291-297.  

 

This 23-year-old retrospective cohort study of EM residency graduates from 1978-1982 used a mailed (what’s the internet?) questionnaire to identify factors associated with leaving the specialty.  There were 539 responses for a 63% response rate.  Factors associated with remaining in emergency medicine were board certification, working with residents during their EM practice, as well as having an income of more than 100K/year.   Ten-year professional survival rate was 85%.  EM physicians who had left the field were more likely to be board certified in another specialty, and less satisfied with EM as their initial choice of specialty.

A reminder of how far we’ve come.  Less than 40 years ago, during these 4 early years of our specialty, there were a total of only 858 EM residency graduates.  Significant numbers of EM physicians were board certified in another specialty.  It’s difficult to know the reasons those early emergency physicians chose training in EM, and there was no such thing as a medical school rotation in EM.  As of 2014, there were 170 emergency medicine programs, with 1,786 EM residency positions offered in the match.   We have 27 types of EM fellowship training, with EM residency groups and faculty advisors to help guide medical students in their choice of career.   An EM physician will become president of the AMA in June of 2015.

 

The first sentence of the article’s discussion is telling:  “Previous reports have questioned the ability of emergency medicine to survive as a specialty due to the amount of stress this type of practice generates.”  While no one would deny the stress of our jobs, our specialty has grown and thrived.  It would be hard to extrapolate findings from this study to today’s population of emergency physicians.

 

 

3.     Wang X, et al:  Fruit and vegetable consumption and mortality from all causes, cardiovascular disease, and cancer: systematic review and dose-response meta-analysis of prospective cohort studies.  BMJ  2014;349:g4490.   

 

“Eat Food, Not Too Much, Mostly Plants.”  

–Michael Pollan, author of The Omnivore’s Dilemma and In Defense of Food, among others

 

In this stunning meta-analysis of 16 articles including >800,000 subjects and 4.6-26 year followup, it was revealed that your mother is right-eat your fruits and vegetables.  Each additional daily serving of fruits or vegetables was associated with a decrease in all cause and CV mortality.  No change in cancer mortality-perhaps because cancer is so heterogenous in origin depending on organ, with potential for substantial genetic predisposition?  Limitations include the self-report nature of surveys and poor adjustment for other dietary factors.  Not a strong article; as Harwood mentioned, we might not be nodding in agreement so readily if it was advocating cobra venom rather than something so aligned with our common sense....but hey, go eat a rambutan.

 

If you’d rather not read the meta-analysis, suggest reading something by Michael Pollan (http://michaelpollan.com/books/).

 

4.     Cramer, H, et al: Effects of yoga on cardiovascular disease risk factors: a systematic review and meta-analysis.  Int J Cardiol  2014 May 1;173(2):170-83. 

 

Finally, an article after my own heart chakra.  A systematic review of 44 RCTs with >3,000 patients, evaluating the effects of yoga on modifiable cardiovascular risk factors.  Small improvements in BP, HR, RR, waist circumference, cholesterol and HbA1c were noted.  Not sure if any of these differences would be considered clinically relevant, and there was high or unclear risk of bias in most studies.  Many different types of yoga were studied.  In the end however, when yoga was present, even if only for weeks, there were positive and beneficial effects noted in multiple modifiable CV risk factors.

Weekly Wellness 2.22.2015

NUTRITIONAL BYTES

Article w/ Recipes: Salads Don't Have to Be Green

 

WORKOUT OF THE WEEK: TRALE RUN

Special shout-out to my man and successful procreator-in-chief Trale "Trale Bear" Permar for clueing me in to an amazing workout opportunity right in our South Side backyard! The Swallow Cliff Stairs are a mere 20 min from our hospital, and provide endless fitness opportunities. With over 125 stone stairs built into the side of a steep cliff, you will not have a problem getting your heart rate up! I went after a 6a shift to check out the stairs, and was able to complete 6 up & down circuits before the wind chill and my wobbly legs forced me back into the car. So do yourself a favor and go to Swallow Cliff and get going! You can walk the stairs, run them, throw some weights in a backpack and train like a mountaineer, or make up your own variations! Before you know it, you'll be as strong as a Trale Bear! 

 

Directions to Swallow Cliff Stairs

 

Videos:  Swallow Cliff Stairs 1

             Swallow Cliff Stairs 2

             How to use the stairs more efficiently

             Stairs with Tony Horton (of P90X fame)

             Just climbing or hopping the stairs too boring for you? Check out these videos for ideas and               inspiration:

             Savage Stairs

             Stairway to Strength

             Stair Workout for Women (Or anyone, actually!)

             Eccentric Stair Workout

 

Bonus Article: To Jump-Start Your Exercise Routine, Be Mindful          

 

 

Radiologists are human too

An 18 y/o male presented to the ED with chest and back pain that started in the center of his chest, then moved to left scapula. Pain was sharp and worse with deep inspiration. His only associated symptoms was anxiety. No risk factors for heart disease, PE. 

VS: 101 20 136/88 20 99%

PE: normal

EKG: normal

CXR Findings: PA and lateral views of the chest are presented for interpretation. No prior studies

are available for comparison. The heart size is normal. The trachea is midline. The lung fields

are clear.

Impression: Radiographically negative chest.

Pt was discharged, symptoms persisted, so he went to another institution where he was admitted. Why? Look carefully at his cxr below

CXR1.png

As you've probably noticed, the radiologist misread the film- or dictated onto the wrong chart, or made some other mistake that we're all capable of making.  So this case serves as both a reminder to get into the habit of looking at all your own films, and a review of a neat CXR finding.  

pms.png
ptx.png

The patient has a pneumomediastinum   and pneumothorax

deep sulcus.png

with deep sulcus.

Spontaneous pneumomediastium is fairly rare. It occurs when air leaks out of a ruptured alveola and tracks along the bronchovascular sheath until it reaches the mediastinum. Air can also track to the pleural space, as in this case, or into the subQ tissues, submandibular and retropharyngeal space, or even into the spinal canal (see Dr. Permar for more information on this complication). Triggering factors can usually be identified but  are so common (crying, Valsalva, coughing, vomiting) that they don't rule in the disease.

The most common presenting symptoms will be pain wherever the air has tracked. Physical examination will likely be normal with isolated pneumomediastinum, although you may be lucky enough to hear Hamman's  crunch (crepitus heard over the heart). You may feel subcutaneous emphysema if the air has tracked there. Patient's with large pneumothoraces may have decreased breath sounds or hypoxia. 

Spontaneous pneumomediastinum is generally a self-limited disease without serious sequelae- much different than if caused by Boerhaave's or trauma. It requires no treatment except analgesia as needed. This patient was admitted for treatment of his pneumothorax.

Weekly Wellness 1.28.2015

NUTRITIONAL BYTES

Article: Omega-3s & Risk of Premature Death

Video: Fish, Mercury, & Nutrition: The Net Effect

Recipe: Halibut with Citrusy Tomatoes and Capers...don't be afraid to substitute the halibut (which is pretty pricy) for another type of white-fleshed fish, such as cod, grouper, snapper, or whitefish)

 

WORKOUT OF THE WEEK

Article: Why Your Workout Should Be High Intensity

 

Workout: "The Toaster"

This is all burpees, baby! Here's some more burpee love from Funk Roberts.

I finished this bad boy in 8'35" and got my HR up to 99% of my calculated max. The shorter you can keep the rests, the more intense it will be!

 

10 burpees...rest (max 60 sec)

5 burpees...rest (max 45 sec)

2 burpees...rest (max 30 sec)

10 burpees...rest (max 60 sec)

5 burpees...rest (max 40 sec)

3 burpees...rest (max 35 sec)

10 burpees...rest (max 60 sec)

5 burpees...rest (max 30 sec)

4 burpees...rest (max 20 sec)

10 burpees...rest (max 60 sec)

5 burpees...rest (max 10 sec)

5 burpees

fin

Transfusion in Critical Illness

Thanks to Dave and Heather Collins for a highly entertaining evening, and to J Beckemeyer, Catherine, Elise, Erin, J Remke and Dr. Febbo for their erudite presentations.

Increasingly, data support a more restrictive approach to blood transfusion.  Think of a blood transfusion as a liquid organ transplant, with added risks of TACO, TRALI and other transfusion related immunomodulation. 

BOTTOM LINE FOR JOURNAL CLUB:  7 is the new 10, and 1 is the new 2.  Transfuse at a hemoglobin of 7 g/dl unless unstable/actively hemorrhaging or having ACS, and give 1 unit of pRBCs at a time.  For ACS, current literature is inconsistent but reviewed article favored the more traditional approach of transfusing to 10 g/dl. 

1.     Villanueva C, et al. Transfusion Strategies for Acute Upper Gastrointestinal Bleeding. N Engl J Med 2013; 368:11-21.

In this single center RCT of 921 patients with acute upper GI bleeding, half were randomized to a restrictive strategy (transfuse when hemoblobin < 7 g/dl) and half to a liberal strategy (transfuse when hemoglobin < 9 g/dl).  Patients were included if >18 yo and evidence of acute upper GIB by hematemesis, melena, or both.   Patients with massive exsanguinating bleeding, ACS, symptomatic PVD, and TIA/CVA were excluded.  They also excluded patients with Rockall scores of zero and hemoglobin >12 (wimpy bleeds).  Randomization was stratified according to presence/absence of cirrhosis.  Transfusions were given 1 unit at a time, with reassessment.  Transfusion could also be given for symptomatic anemia, massive bleeding, or need for surgical intervention.  All patients underwent endoscopy within 6 hours.

Primary outcome of survival at 6 weeks was significantly higher in the restrictive (95%) versus liberal (91%) group.  Risk of further bleeding, adverse events, and rescue therapies were significantly lower in the restrictive group. Patients in the liberal group also had significant early rises in portal-pressure gradient (increases risk of rebleeding).  The only subgroup without a significant mortality difference were patients with advanced cirrhosis.

Why does restrictive strategy work in GI bleed?  Authors posit that transfusion may counteract protective splanchnic vasoconstrictive response caused by hypovolemia.  This increased pressure may impair clot formation.  Transfusion may also induce coagulation abnormalities, and for patients with cirrhosis, increased blood volume can increase portal pressure leading to rebleeding.

BOTTOM LINE:  great study looking at a population that makes us nervous, supporting a restrictive transfusion strategy to improve mortality, decrease rebleeding/adverse events, and conserve resources.  It’s a win/win, but remember that they excluded patients with exsanguinating bleeding as well as those with ACS.  Also, transfusion could be given for symptomatic anemia, massive bleeding, or if need for surgery-important caveats.  Finally, these patients all underwent early endoscopy, facilitating early intervention for ulcer, varices, etc.

 

2.     Holst LB, et al:  Lower versus higher hemoglobin threshold for transfusion in septic shock.  N Engl J Med  2014 9;371(15):1381-91. 

In this multicenter, randomized trial, 998 ICU patients with septic shock were randomized to a lower threshold group (transfuse 1 unit pRBCs for hemoglobin < 7 g/dl) or higher threshold group (transfuse 1 unit pRBCs for hemoglobin < 9 g/dl). 

Patients with stable cardiovascular disease were included, but patients with life threatening bleeding or ACS were excluded.  Patients developing life threatening bleeding, ischemia, or need for ECMO or surgery after enrollment could receive transfusion at discretion of treating physician.  Leukoreduced blood was used in an attempt to mitigate immunomodulatory effects of transfusion.

Primary outcome of death by 90 days occurred in 43% of lower threshold group compared with 45% of higher threshold group (RR 0.94; 95% CI 0.78-1.09).  Rates of ischemic events, severe adverse reactions, and life support requirements were also similar in the two groups.  The lower threshold group received half the total number of transfusions as the higher threshold group.  It’s an ICU, not an ED study, but given our boarding, probably still pretty useful information.

BOTTOM LINE: in ICU patients with septic shock, similar mortality and adverse event rates were observed with a lower transfusion threshold (transfuse for hemoglobin < 7 g/dl), with significant savings in the number of units of blood transfused.

 

3.     Carson JL, et al: Liberal versus restrictive transfusion thresholds for patients with symptomatic coronary artery disease. Am Heart J  2013;165(6):964.

Ok, so clear that in GI bleed and sepsis (and from the unfortunately named TRICC trial, for critically ill patients in general) a more restrictive blood transfusion strategy is safe and effective.  The holy grail remains...what about patients with ACS??

In this “pilot trial” of 110 patients with ACS or stable angina undergoing cardiac cath, patients were randomized to receive transfusion for hemoglobin < 10 g/dl (liberal strategy), or to transfusion for symptomatic anemia as well as permitted/not required to receive transfusion for hemoglobin < 8 (restrictive strategy).

Patients with active bleeding, hemodynamic instability, symptomatic anemia at time of randomization, or needing surgery were excluded.

Primary outcome was the composite of death, MI, or unscheduled revascularization 30 days after randomization, and occurred in 11% of liberal group and 25.5% of restrictive group (95% CI 0.7% – 29%).  Death at 30 days was 2% in liberal group vs. 13% in restrictive group (95% CI 1.5% – 21%).  These were not significant differences due to small group sizes/large CI, but trends favored liberal transfusion strategy.  All deaths were classified as cardiac.  Most other adverse cardiac outcomes were more frequent in restrictive group.

This study had problems.  First, it’s a small study, so can’t draw practice-changing conclusions.  Also, the restrictive group was on average 7 years older than the liberal group.  Patients with ACS are different from those with stable disease undergoing cath-really these are 2 separate populations.  The study was terminated prior to completing their planned enrollment of 200 patients...not clear why...they do some hand waving about having enough information to plan a larger trial.  Interestingly, the most frequent reason for protocol violation was insufficient time to administer transfusion prior to discharge in the liberal group (wait Mr. Smith...don’t call a taxi yet, let’s give you an [unnecessary?] transfusion).

BOTTOM LINE:  Not a great study, but trends support a more liberal strategy (transfuse for hemoglobin < 10) in patients with ACS. Existing literature for this population is inconsistent, equipoise exists, and a large, well done study still needs to be performed.

Weekly Wellness 1/17/2015

Hello Christ Docs,

Happy New Year! Weekly Wellness is back for 2015! Just a reminder, my goal is to encourage healthy lifestyle habits by of providing interesting articles on topics covering nutrition and fitness, as well as give you ways to live a more healthy lifestyle with delicious recipes and workouts you can do at home in less than 20 minutes with no special equipment.

I am always interested in learning more, so if you have any great recipes, know a good article, or have a workout you'd like to recommend, please pass it along!

Keep making healthy choices when you can!

Bret

 

NUTRITIONAL BYTES

Article: Why Eating Seasonally Is Good For You

Recipe: Chicken Paillards with Squash & Spinach 

             Just made this last night, and oh boy was it good! You can use any kind of squash, (I used acorn), but I would roast the squash & red onions for double what Martha recommends (30 min vs 15 min). The flavors are awesome (don't skimp on the sage), and its healthy and lean and pretty darn easy! I used a rolling pin to flatten my chicken breasts. 

 

WORKOUT OF THE WEEK 

Articles: How Exercise Keeps Us Young

Workout: 

Throw on some motivational music, and give this a whirl. Don't stop the clock for breaks. My time was 14:14, and I got my heart rate up to 97% of my maximum calculated. Calculate your heart rate here, and see how close to your max you can get!

 

10 ROUNDS FOR TIME

10 Pushups

10 Crossfit Situps (Don't worry about not having an AbMat...I used a folded thin yoga mat or blanket to get the lumbar support)

10 Airsquats (do JumpSquats for added challenge, or, if you are really intense, do JumpSquat Tuck Jumps!)

 

chest pain

36 yo M, no pmhx, appears morbidly obese, no hx of HTN,but  has been told once he has borderline HTN, takes no meds. Presents by car with his father.  Patient states he had sudden onset left upper chest pain that radiated to his neck with mild sob and mild lightheadedness that started when he was walking up stairs.  He states the sob and lightheadedness have resolved and now the chest pain is mild.  It did not radiate to his back.  Nonexertional, nonpleuritic.  No hx of DVT or PE, no leg swelling or pain.   Denies abd pain.  Denies diaphoresis or nausea.  Pt is a smoker.  Denies drug use, admits social alcohol use.  Denies Fhx of early heart disease or suddent death.

PE is unremarkable

Vitals:  Hr 110, BP 122/84, RR 20, RA P02 97%, Temp 36.3

EKG: NSR no acute st or t wave changes HR 110

Inital A/P:

Well appearing morbidly obese patient with exertional onset left chest pain radiating to left neck, no radiation to back.  Denies medical hx and follows w a primary doctor.  Normal PE, no risk factors for PE.  No fmhx of CAD.  Non ischemic ekg and pain is currently mild.  He did state his pain was moderate to severe at onset.

Planned DDimer, trop x2, CXR, labs, and entering the patient into CPEP protocol with planned AM stress cardiac echo

Patient initially refused admission but was talked into staying for the protocol. wow

Results:

Tropx2 negative

DDimer negative

2 view CXR negative

CBC CMP normal

repeat ekg - non ischemic tachycardia resolved

At 7am near the end of signout I was called by the nurse as the patient just vomitted and BP was 100/55, HR was still 70s.

I reevaluated the patient and he appeared well and stated his pain returned and was still mild left CP radiating to his neck.   No other complaints besides nausea.

At this point I figured he couldnt do the stress test and changed him to an admission.  I decided to reevaluate if there was anything I was missing.  

Due to my past mental trauma from seeing 7 total patients younger than 45yo with aortic dissection in 7 years of residency/practice and 1 of those patients dying from their dissection.....I decided if I was to get any sleep I would need to get a CTA chest abd pelvis w and wo contrast prior to admission to tele.  The tipping point in my mind was why was this guy borderline hypotensive if he is in pain and morbidly obese.  Even with only moderate pain without alot of the classic symptoms of dissection.  He even had bilateral BPs done by accident which were not marked abnormal.  Also normal 2V chest and normal DDimer.  All prior 7 dissections in young people that I had seen had elevated DDimers.  It is NOT a sensitive test but with very low suspicion...sometimes to me it is somewhat reaasuring....THere is much debate about it's utility.

830am CTA results:

Large aneursymal dilatation of the proximal aorta and aortic valve.  Largest aortic diameter is 6cm.  Cannot rule out dissection within the aneursym.  Moderate size pericardial effusion vs pericardial hematoma.

Cardiothoracic surgeon immediately paged.  Cards came to the bedside.

11am Bedside ECHO results:

Large aneursymal dilatation of the proximal aorta and aortic valve.  Largest aortic diameter is 6cm.  Cannot rule out dissection within the aneursym.  Moderate size pericardial effusion vs pericardial hematoma with diastolic collapse of the RV concerning for possible early tamponade.

1pm in the OR

TEE confirms a dissection just above the aortic valve within the aneurysm.  Moderate aortic regurgitation. Moderate to severe pericardial effusion.

Cocaine negative on UTox

In the OR:

Patient had cardiac bypass/open chest sugery with replacement of ascending aorta w graft and his aortic valve was repaired but not replaced.

Post op day #1 the patient was extubated and stated he felt fine except for some incisional pain.

WOW...dodged a bullet

Almost gave this guy a stress test.  He didnt even want to stay for that.  Thankfully he vomitted and had borderline hypotension and thankfully I have a history w dissection that has made me a little more cautious....or I would have missed this.   Could've been my second death from a missed dissection...probably on the tredmill in the stress lab.  The prior death of a patient I had from dissection was a 33 yo M with a similar presentation and workup as this patient.  I discharged him and he came back on my same shift and died within 30 of arrival.  

Normal BP on arrival.  Normal bilateral BPs.  Normal 2V CXR.  negative trops and ddimers.....recipe for disaster.

Thanks to the resident and the next attending for fully taking over this guy at 1030 am and making sure he got the ECHO and the OR.  

Take homes:

- Morbid Obesity is a risk for other comorbidities (undiagnosed HTN) and dissection.  None of my young dissections appeared marfanoid but almost all were obese with probable undiagnosed or poorly treated HTN

Kyobu Geka.

 2013 Jun;66(6):437-44.

[Obesity is a risk factor of young onset of acute aortic dissection and postoperative hypoxemia].

[Article in Japanese]

Aizawa K

1, 

Sakano Y

Ohki S

Saito T

Konishi H

Misawa Y

.

- He was never truly hypotensive but borderline hypotension in a patient you would think would be hypertensive with chest pain should raise some eyebrows

- DDimer has been argued by some as a screening test for dissection but it's not sensitive enough.  This case illustrates that.  I feel that you could miss fewer dissections by using it w risk stratification, normal pulses in all four, normal 2V cxr, normal bilateral BPs etc, but if you have clinical suspicion...you have to scan

- Consider doing some documentation with chest pain patients that would make your chart more defensible in case you miss AD.  They are easy to miss and I have been there - things you could add would be equal pulses in all 4 ext, equal bilat BPs, normal CXR, and reasoning that you think the probability of dissection is so low that CTA is not indicated.

- Some experts on aortic dissection argue that its presentation can be so nonspecific that the standard of care is to not make the diagnosis.  

 Below is a CTA image showing the proximal aortic dilation and some evidence of the pericardial effusion

dissection2.jpg